Law & Business Administration

¡Supera tus tareas y exámenes ahora con Quizwiz!

Explain how the introduction of the Charter of Rights and Freedoms increased the protection of human rights in Canada.

Before the introduction of the Charter, human rights were not entrenched in the Constitution. After the introduction of the Charter, the court could strike down government action that did not conform to the Charter. Government could override court decisions regarding human rights only by passing legislation notwithstanding the rights contained in it. The Charter can be amended not by ordinary legislation but only by using the difficult amendment process contained in it. Diff: 2 Type: ES Topic: The Charter of Rights and Freedoms Skill: Recall 61

In law, the concept of negligence continues to evolve, so that what was not negligence in the past may well become negligence in the future. Explain.

Being founded on a duty or standard of care, circumstances that may constitute negligence change with society and what at one time may not have given rise to a duty of care or set a standard of care may well change at another time. Diff: 3 Type: ES Topic: The Basis for Liability Skill: Applied 58

Explain how an Internet contract is formed.

Businesses selling online use standard form terms and conditions in their contracts. E-commerce legislation has modernized contract formation rules so that "clicking an icon" satisfies the acceptance and communication requirements of contract formation. In an Internet contract, the Internet consumer is the offeree, and after seeing the terms required to be disclosed by the retailor (offeror

What do we mean when we say that the two levels of government have concurrent powers?

Concurrent powers describes the situation when, under the constitution, both the federal and provincial governments appear to have the same authority to regulate a given activity. Diff: 2 Type: ES Topic: The Courts and Legislation Skill: Recall 51

There are five distinct steps in developing a legal risk management plan. List these steps.

Conduct a legal audit of the business; prioritize the risks; develop a strategy to address each risk; implement the plan; and regularly review and update the plan. Diff: 2 Type: ES Topic: Legal Risk Management Skill: Recall 49

Motive is irrelevant to consideration. Explain.

Consideration is the price that one person pays for the promise of another, or something that is given in exchange for the promise of another. This is very different from the reason behind the making of the promise. The reason why Jack makes a promise to pay James $40.00 in exchange for James' promise to wash Jack's clothes is irrelevant. Only consideration is relevant in the eyes of the law. Diff: 1 Type: ES Page Reference: 116 Topic: Motive Contrasted with Consideration: Past Consideration Skill: Recall 46

What do we mean when we say that damage is remote?

Damage or injury suffered by a person is said to be remote when it could not have been foreseen by the wrongdoer as likely to result from her or his acts or omissions. Diff: 2 Type: ES Topic: Unintentional Torts Skill: Applied 67

What is deceit?

Deceit is the intentional making of a false statement with the intention of misleading another person. Diff: 1 Type: ES Topic: Other Torts Skill: Recall 42

Explain why it can be important in terms of enforcing a contract to ensure that you have all the licences required by government regulation and by your trade.

If you do not have the required licences and you are suing to enforce the contract, the defendant may be able to defend on the basis of illegality. You would then not be able to recover compensation for the services provided. It may be possible in some cases to recover for materials supplied. Diff: 2 Type: ES Topic: Contracts Affected by Statute Skill: Recall

In Canada, law comes from a variety of sources. State each source and explain what type of law comes from it.

In Canada, our law comes from the Constitution, which is the basic law from which all other laws draw their power. After the Constitution, the law comes from legislation passed by both the federal and provincial governments, as well as subordinate legislation called regulations, which are passed by a specific body referred to in that legislation, and decisions made by administrative tribunals set up by the legislation. Finally, law comes from decisions of the courts, based on principles of law that have been developed in earlier court decisions and to which we refer as case law. Diff: 2 Type: ES Topic: Who Makes Law? Skill: Applied 59

Explain how a bilateral contract differs from a unilateral contract.

In a bilateral contract, both parties trade promises and an offeree accepts and communicates such acceptance to the offeror. In a unilateral contract, only the offeree makes a promise to all. A contract arises when a person, the offeree, steps forward to perform the contract. There is no communication of acceptance by an offeree in a unilateral contract. Diff: 1 Type: ES Topic: Unilateral and Bilateral Contracts Skill: Applied 48

In a federal country such as Canada, the Supreme Court often has the last word. Explain.

In a federal country such as Canada, there are two levels of government, the national government and the provincial governments. Pursuant to the Constitution Act, 1867, each level of government has an independent existence and its own sphere of activity. There are also areas where the two levels of government have concurrent jurisdiction. When problems arise with respect to jurisdiction, it is the Supreme Court of Canada that makes a final determination as to the interpretation of the law and the exercise of jurisdiction. Diff: 3 Type: ES Page Reference: 10 Topic: The Courts and Legislation Skill: Recall 54

Damages for breach of fiduciary duty and breach of contract are based on the same principle. Explain.

In both breach of contract and breach of fiduciary duty, damages are intended to be restitutionary. In both situations, damages are intended to place the party in the place he or she would have been in had the breach not occurred. The characterization of a claim as a breach of fiduciary duty will not entitle a plaintiff to a higher award. Diff: 3 Type: ES Topic: Liability of Professionals Skill: Applied 54

Canada is a federal system. Briefly explain what this means.

In the Canadian federal system there are two levels of government: the Federal government, or Parliament, and the Provincial governments. The power is shared between each under sections 91 and 92 of the Constitution Act. Diff: 2 Type: ES Topic: The Courts and Legislation Skill: Applied 50

Explain the development of fault as an appropriate basis for liability in tort.

In the early development of tort law, liability was strict. There was no inquiry into the reasons for the injury and whether the wrongdoer's conduct was justified. However, the idea developed that a person ought not to be liable for injury that he or she did not cause. Thus, tort law soon began to take into account the fault of the alleged wrongdoer in causing injury. Diff: 3 Type: ES Topic: The Basis for Liability Skill: Applied 62

What is the difference between intentional torts and negligence?

Intentional torts involve behaviour done deliberately that causes injury or loss to another person. Negligence is an unintentional tort, usually the result of careless activity on the part of a defendant. Negligence traditionally requires the court to consider whether the defendant's conduct fell below a reasonable standard of care in all the circumstances. Diff: 3 Type: ES Topic: Torts Skill: Applied 72

Is it possible for a person to enter into an enforceable contract without receiving any benefit from the agreement?

It is possible for a person to enter into an enforceable contract even if that person does not receive any benefit from the agreement. The only legal requirement regarding consideration is that it must be of some value. Thus consideration may be both a benefit and a detriment. For instance, a party, A, enters into a contract with another, B, on terms that A will supply goods to B in payment for property of B's destroyed by A, if B does not bring proceedings against A. A receives no benefit from the contract, and yet an enforceable contract is created. Diff: 3 Type: ES Topic: Adequacy of Consideration Skill: Recall/Applied 50

How can the plaintiff's own conduct affect the outcome of a claim for negligence?

Jurisdictions have contributory negligence legislation that requires the courts to apportion damages according to the degree of responsibility of the parties. If a plaintiff is injured but contributed to the injury through the plaintiff's own fault, the compensation that the defendant is required to pay will be reduced. The courts apply a "comparative blameworthiness" approach in apportioning responsibility. Diff: 2 Type: ES Topic: Unintentional Torts Skill: Recall 78

Explain, with the use of examples, the concept of legal capacity.

Legal capacity means competence to bind oneself. Thus a person is said to have legal capacity if the person is competent to enter into a contract and bind him- or herself. For example, a minor, that is, a person who has not reached the age of majority, is not competent to bind him- or herself because a minor is deemed to have a weak bargaining position. As well, certain persons of unsound mind are also deemed to lack legal capacity. Diff: 2 Type: ES Topic: The Meaning of Capacity to Contract Skill: Recall/Applied 46

Explain, with the use of examples, the concept of legal capacity.

Legal capacity means competence to bind oneself. Thus a person is said to have legal capacity if the person is competent to enter into a contract and bind him- or herself. Thus a minor, that is, a person who has not reached the age of majority, is not competent to bind him- or herself because a minor is deemed to have a weak bargaining position. As well, certain persons of unsound mind are also deemed to lack legal capacity. Diff: 2 Type: ES Topic: The Meaning of Capacity to Contract Skill: Recall 54

, not every professional relationship will be fiduciary in nature. Explain.

Many types of relationships involving professionals may not necessarily be fiduciary in nature. It will depend on the particular facts of the case. In order to establish a fiduciary relationship, five factors must be considered carefully: the vulnerability of the client; whether the relationship involved trust of the professional; whether the client relied (to the client's detriment

Diminished capacity for minors is not treated in the same way as it is for drunken or insane persons. Explain.

Minors can establish diminished capacity simply by proving their age, while drunken or insane persons must establish that not only were they incapacitated by alcohol or mental infirmity at the time of the contract, but also that the other party was aware of their condition. Diff: 2 Type: ES Topic: Other Persons of Diminished Contractual Capacity Skill: Recall 44

Critics of class actions say that the legislation encourages frivolous lawsuits that are expensive to defend. Explain the benefits of class actions.

A class action allows one individual to represent a group or class of others in one proceeding. Multiple actions and inconsistent results are thereby eliminated. Furthermore, any settlement or judgment binds all members of the class as well as all defendants. This ensures that a subsequent action cannot be brought before the court to contest liability. Finally, class actions can increase access to the courts by allowing claimants to come forward with claims of a small value that might previously have been too costly to litigate. Diff: 3 Type: ES Topic: Procedural Law: Using the Courts Skill: Applied 1

Explain how a confidentiality provision in an employment agreement can give rise to an ethical dilemma for an employee and whether the law should permit the breach of such a provision where it is warranted.

A confidentiality provision is one that prohibits the employee from disclosing confidential information gained in the course of employment. Sometimes, however, an employee will come into confidential information that suggests that the practices of the employer create a danger to the public. If the employee were to disclose this information, the employee would be committing a breach of the confidentiality provision, giving rise to sanctions against the employee by the employer, such as termination of employment, a law suit in damages, or both. Yet one can argue that the employee has a higher duty to the public to disclose information that, if disclosed, could prevent injury or harm to the public. In this latter sense, the courts should acknowledge this higher duty and permit the disclosure of such information. Diff: 3 Type: ES Topic: Contracts Illegal by the Common Law and Public Policy Skill: Recall/Applied 52

What is a conflict of interest?

A conflict of interest is a situation where a person's duty conflicts with his or her interests, for example, where a person owes a duty to two or more persons with competing interests. His or her interest in making money by representing both persons will conflict with his or her separate duty of loyalty to each of them. Diff: 1 Type: ES Topic: Liability of Professionals Skill: Recall 48

All contracts begin with a promise, but not all promises become contracts. Explain.

A contract is a legally binding promise. A legally binding promise is a promise that gives rise to a contract. Where a promise is not legally binding, for example, it is gratuitous, it will not give rise to a contract. The law will not enforce a gratuitous promise. Diff: 1 Type: ES Topic: The Nature of a Contract Skill: Applied 51

Explain the role of legality in the formation of a contract.

A contract must be legal, that is, it must not offend the public good and not violate any law. In the absence of any evidence to the contract, the court presumes that all transactions are legal. A defendant must adduce evidence to establish that this is wrong. Diff: 2 Type: ES Topic: The Role of Legality in the Formation of a Contract Skill: Applied 56

What is a counter-offer? What effect does a counter-offer have on an offer?

A counter-offer is made when an offeree responds to an offer by indicating his or her willingness to enter into a contract with the offeror, but on terms that are different from the offer. A counter-offer, when made by an offeree, amounts to a rejection of the earlier offer and the making of a new offer by the offeree, who now becomes the offeror. Diff: 1 Type: ES Topic: Rejection and Counter-offer by the Offeree Skill: Recall 52

Explain what a court of first instance is.

A court of first instance is a trial court where witnesses give evidence and an initial judgment is made. The topic of dispute will determine in which trial court the dispute belongs. Diff: 1 Type: ES Topic: The System of Courts in Canada Skill: Recall 52

What is a gratuitous promise? Can such a promise support a contract?

A gratuitous promise is made without bargaining for, or accepting, anything in return. A gratuitous promise goes against the general rule that a promise must be supported by consideration to be enforceable. Thus, as a general rule, a gratuitous promise cannot support a contract. The exception concerns those situations characterized as situations involving injurious reliance, where a party has relied on a gratuitous promise to his or her detriment. Where a party relies on a gratuitous promise to his or her detriment, the promisor will not be permitted to renege on the promise. Diff: 2 Type: ES Topic: Gratuitous Promises Skill: Applied 48

Explain what is meant by the liberal approach taken by judges in interpreting statutes.

A liberal approach involves consideration of the context, the custom, and trade usage of the language, as well as the intent or purpose of the government when it passed the law. This will involve looking at the state of the law at the time of the passage of the statute and the language used when the bill was introduced and debated. Diff: 2 Type: ES Topic: Legislation: Government Made Law Skill: Recall 50

Explain how a warning issued by a manufacturer to a "learned intermediary" discharges the duty of the manufacturer to warn consumers of defects in a product.

A manufacturer has a continuing duty to warn consumers of dangers and defects associated with products it places on the market. The manufacturer may issue the warning to a learned intermediary, namely, a person whom the consumer consults about the use of the product—for example, a doctor. The duty to warn is discharged if the warning is issued to a learned intermediary. Diff: 3 Type: ES Topic: Unintentional Torts Skill: Recall/Applied 69

The duty of a manufacturer to warn may be discharged by issuing a warning to learned intermediaries. Explain.

A manufacturer has a duty to warn consumers after its products are placed on the market. This duty extends to warnings regarding the improper use of the product. However, if the manufacturer issues warnings to learned intermediaries, such as doctors, on how to use the product properly, the manufacturer's duty towards the consumer is discharged. Diff: 2 Type: ES Topic: Product Liability Skill: Applied 61

Explain the concept of absolute privilege and its application.

Absolute privilege provides complete immunity from liability for defamation, but is restricted to defamatory statements made during parliamentary debate, in proceedings in law courts and inquests, and before royal commissions. Diff: 2 Type: ES Topic: Intentional Torts Skill: Recall 59

Why is a court more likely to accept as reasonable certain restraints on an employer who is in possession of certain trade secrets of his or her employer?

Agreements between employer and employee may restrict the future economic freedom of the employee. For example, an agreement may restrict the employee from working in the same kind of employment or for a competitor after a period of time when he leaves the employ of his or her current employer. A court is more likely to see as reasonable restrictions that concern employees who are in possession of trade secrets or who have direct dealings with the employer's clientele. This is because of the perception of the court that there is a more urgent need to protect business and other proprietary secrets of the employer. Diff: 2 Type: ES Topic: Agreements in Restraint of Trade Skill: Recall 55

Why is a court more likely to accept as reasonable certain restraints on an employer who is in possession of certain trade secrets of his or her employer?

Agreements between employer and employee may restrict the future economic freedom of the employee. For example, an agreement may restrict the employee from working in the same kind of employment or for a competitor after a period of time when he or she leaves the employ of his or her current employer. A court is more likely to see as reasonable restrictions that concern employees who are in possession of trade secrets or who have direct dealings with the employer's clientele. This is because of the perception of the court that there is a more urgent need to protect business and other proprietary secrets of the employer. Diff: 2 Type: ES Topic: Agreements in Restraint of Trade Skill: Recall/Applied 51

Explain the principle of vicarious liability.

Although an employee may be primarily liable for a tort, if the tort is committed in the course of employment, the employer is also liable in a secondary sense. Diff: 2 Type: ES Topic: The Basis for Liability Skill: Recall 57

Even a trespasser is owed a duty of common humanity. Explain.

An occupier or owner of premises owes a minimal duty to a trespasser. He or she must not deliberately set out to harm the trespasser or recklessly disregard the possibility that his or her acts might harm the trespasser. Diff: 2 Type: ES Topic: Occupier's Liability Skill: Applied 56

What is the legal effect of a counter-offer?

An offer is a tentative promise made by one party, subject to a condition, or containing a request to the other party. An offer must be communicated to the other party before it can be accepted. In most situations where parties bargain, an offer that is made is not accepted immediately but is followed by another offer by the other party. For instance, if Jack says, "I would like to sell the car to you for $5000" and Mary says, "I will pay only $4000," Mary does not accept the offer of Jack but makes another offer. Mary's offer is known as a counter-offer. The legal principle is that when a party makes a counter-offer, it amounts to a rejection of the offer and the substitution in its place of a new offer from the other party. Diff: 1 Type: ES Topic: Rejection and Counter-offer by the Offeree Skill: Recall 53

Explain the difference between an invitation to do business and an offer.

An offer to make a contract can be accepted, and when it is accepted, this gives rise to a contract. However, an invitation to do business is not an offer that is capable of being accepted. It is an invitation to enter into negotiations that could result in a contract. An example of an invitation to do business is an advert in a store that advertises products for a particular price. When a person picks up a product in a store, he or she cannot insist on purchasing it for the price in an advert because the advertised price is not an offer. Diff: 1 Type: ES Topic: The Nature of an Offer Skill: Applied 49

Deciding on the type of action to bring for acts or omissions of a professional who breached a duty of care was not always easy, but this has changed. Explain.

At one time considerations such as the date the cause of action arose and the nature of damages that might be awarded were factors in deciding whether to sue in contract or in tort. However, today this is overcome by suing in both contract and tort. Diff: 1 Type: ES Topic: Liability of Professionals Skill: Applied 43

, is one of the major shareholders of Canco. This situation could result in a a. violation of the statute. b. breach of duty. c. conflict of interest. d. fiduciary duty. e. breach of contract.

c Diff: 1 Type: MC Topic: Liability of Professionals Skill: Recall 18

A duty to account is a. the duty of the court to examine the accounts of a professional who is accused of having misappropriated funds. b. the duty of a professional to submit timely and accurate bills to the client for payment. c. the duty owed by a person who commits a breach of trust to hand over profits made as a result of the breach. d. the duty of a client to pay bills that are rendered by the professional. e. the duty of the professional to maintain accurate accounts of the invoices submitted to his or her client.

c Diff: 1 Type: MC Topic: Liability of Professionals Skill: Recall/Applied 28

During the early development of the common law, whether an occupier or owner of premises owed a duty of care to persons who came on the premises and the standard of care owed depended on whether the persons were invitees, trespassers, or a. children. b. public officials. c. licensees. d. law enforcement officials. e. thieves or burglars.

c Diff: 1 Type: MC Topic: Occupier's Liability Skill: Recall 17

The greater exposure of professionals to liability for professional negligence has led to the extensive use of a. private investigators. b. acceleration clauses. c. liability insurance. d. disclaimer clauses. e. professional fees.

c Diff: 1 Type: MC Topic: Professional Liability: The Legal Challenges Skill: Recall 26

The two main sources of law are a. common law and canon law. b. subordinate legislation and judge-made law. c. judge-made law and statute law. d. judge-made law and canon law. e. judge-made law and equity.

c Diff: 1 Type: MC Topic: The Sources of Law Skill: Recall 21

When a contract is authenticated by the affixing of a seal, the contract will be enforceable even when it lacks a. a promissory basis. b. an acceptance. c. consideration. d. contractual intention. e. an offer.

c Diff: 1 Type: MC Topic: The Use of a Seal Skill: Applied 17

Which of the following is NOT true about consideration? a. Consideration must be of some value in the eyes of the law. b. A gratuitous promise is not supported by consideration. c. Consideration has to be adequate. d. Consideration must be sufficient. e. Consideration can be in the form of a detriment to a party.

c Diff: 2 Type: MC Topic: Adequacy of Consideration Skill: Applied 8

The rule stare decisis a. is a hard and fast rule that has qualifications attached to it that allow judges to depart from it. b. is merely a principle that applies in those countries where the law is codified. c. is not a hard and fast rule and is not always followed. d. is a hard and fast rule that is always followed. e. has no place in a system based on common law.

c Diff: 2 Type: MC Topic: Common Law: The Theory of Precedent Skill: Recall 6

A gratuitous promise differs from a contract in that a. a gratuitous promise cannot lapse. b. it does not require the payment of money to the offeror by the offeree. c. consideration flows only in one direction. d. a gratuitous promise cannot be revoked. e. the terms of a gratuitous promise are not precise enough to be binding.

c Diff: 2 Type: MC Topic: Gratuitous Promises Skill: Recall 4

An intention to create legal relations exists if a. the promisor and the promisee have identified all the consideration involved in the contract. b. there is an offer, acceptance, and consideration. c. a reasonable person would believe that the parties intended to be bound by the contract. d. the court examines the facts and determines that the parties intended to enter into legal relations. e. the parties have capacity to enter into a contract.

c Diff: 2 Type: MC Topic: Intention to Create Legal Relations Skill: Applied 21

During a parliamentary debate, Jack accuses Jim of lying to his constituents Jim decides to sue. Which of the following is true? a. Jack has a defence of absolute immunity. b. Jack has a defence of qualified privilege. c. Jack has a defence of absolute privilege. d. Jim may bring an action for negligent misstatement. e. Jack's conduct is called libel.

c Diff: 2 Type: MC Topic: Intentional Torts Skill: Applied 32

A lawyer who enters into a contact to purchase his or her client's summer cottage has a. lots of money. b. committed a breach of contract. c. a conflict of interest. d. a duty not to bargain with the client for the best price. e. lots of time on her or his hands.

c Diff: 2 Type: MC Topic: Liability of Professionals Skill: Recall/Applied 21

Use the fact situation in Q8 to answer the related question that follows. In the fact situation above, Jonny is a. a trespasser. b. an occupier. c. a licensee. d. a contractual entrant. e. an invitee.

c Diff: 2 Type: MC Topic: Occupier's Liability Skill: Applied 11

James has been burgled several times. He is fed up and decides to do something about it. He places two traps at vantage locations in his home. One night he hears a scream and he runs down to find Bob, a local troublemaker, writhing in pain and holding his bleeding leg. Which of the following is NOT true? a. Bob is not an invitee. b. James owes a duty to Bob. c. James owes no duty to Bob because Bob is a trespasser and was unlawfully on the premises. d. Bob can sue James for deliberately setting the trap to harm him. e. Bob is a trespasser.

c Diff: 2 Type: MC Topic: Occupier's Liability Skill: Applied 18

The normal steps in a civil lawsuit in Canadian courts are a. statement of claim, statement of defence, discovery of documents, pre-trial conference, and trial. b. statement of claim, statement of defence with or without counterclaim, oral and documentary examinations for discovery, pre-trial conference, and trial. c. statement of claim, statement of defence, oral and documentary examinations for discovery, and trial. d. statement of claim, statement of defence, pre-trial conference, and trial. e. statement of claim, statement of defence with or without counterclaim, oral examinations for discovery, pre-trial conference, and trial.

c Diff: 2 Type: MC Topic: Procedural Law: Using the Courts Skill: Recall 14

Which of the following statements is NOT true? a. A manufacturer's duty towards consumers continues after it places a product on the market. b. A retailer may be liable to the consumer for a defective product. c. After a manufacturer places a product on the market, its duty towards consumers ends. d. A manufacturer's duty to warn its customers may be discharged if the warning is issued to a learned intermediary. e. A manufacturer who places a defective product on the market may be liable for negligence.

c Diff: 2 Type: MC Topic: Product Liability Skill: Applied 19

Which of the following is true? a. Where a portion of a contract is void, the whole contract is affected and a court will set aside the whole contract as being void. b. A contract that contemplates the commission of a tort is not legal unless the tort is committed and someone is injured. c. Where a portion of a contract is void, the remainder of the contract is not affected provided a court can sever the void parts without doing injustice to the parties. d. The common law supersedes statute law when determining the legality of contracts. e. A statute can only make a contract illegal; it cannot make a contract void.

c Diff: 2 Type: MC Topic: The Difference Between a Void and an Illegal Contract Skill: Recall 21

James does some work for Jim. Jim refuses to pay on the grounds that not all the work was done. Jack takes Jim to court and the judge says Jack is entitled to a quantum meruit. This means that a. the parties will agree on the appropriate price. b. a dispute regarding the price should be resolved by arbitration. c. Jim must pay a reasonable amount for the services rendered. d. a price will be fixed for the work done. e. Jack is entitled to his expenses.

c Diff: 2 Type: MC Topic: The Effect of a Request for Goods or Services Skill: Applied 16

When a court strikes down an unreasonable agreement between an employer and employee that restricts the future economic freedom of the employee, the court serves two public interests, namely, protecting employees and a. maintaining peaceful workplaces. b. applying the law. c. protecting businesses. d. protecting the mobility of labour. e. protecting the management of businesses.

d Diff: 1 Type: MC Topic: Agreements in Restraint of Trade Skill: Recall 23

Substantive law a. means the civil law. b. comprises law that applies to rulings of judges. c. means the common law. d. comprises the rights and duties that each person has in society. e. comprises laws that are applied to issues or substantive matters that are identified by lawyers in court.

d Diff: 1 Type: MC Topic: Classifying Law Skill: Recall 19

During the early stages of the development of tort law, liability was a. dependent on a finding of guilt. b. punishable by incarceration. c. dependent on whether the wrongdoing was justifiable. d. strict. e. based on a finding of criminal liability.

d Diff: 1 Type: MC Topic: Development of the Tort Concept Skill: Recall 23

Even when offer and acceptance and consideration are present, there may be no enforceable contract if a. the subject of the contract is too trivial. b. there were no witnesses present at the signing of the contract. c. the contract is not in writing. d. there is no intent to create legal relations. e. the consideration is not adequate.

d Diff: 1 Type: MC Topic: Intention to Create Legal Relations Skill: Recall 20

The form of law that was adopted by the whole of continental Europe was one in which a. the law was not codified. b. the law was both codified and based on previous cases. c. the law was based on decisions of previous cases. d. the law was codified. e. the law was partly, but not fully, codified.

d Diff: 1 Type: MC Topic: Legal Systems: Civil Law and Common Law Skill: Recall 3

A fiduciary duty exists between a professional and his or her client when a. the professional lends money to the client. b. the principle of equity imposes such a duty. c. the client owes money to the fiduciary. d. a special relationship of trust exists. e. a contract exists between the professional and the client.

d Diff: 1 Type: MC Topic: Liability of Professionals Skill: Recall 13

Mary is 13 years old, and she enters into an agreement with a modelling agent pursuant to which the agent agrees to represent her. Which of the following is NOT true? a. The agent can only enforce a contract regarding necessaries against the minor. b. A minor may repudiate contracts for non-necessaries because he or she is not bound by them. c. The contract is enforceable against the agent. d. The contract is not enforceable by Mary against the agent. e. The agent cannot enforce the contract against Mary.

d Diff: 1 Type: MC Topic: Minors (or Infants

When a party wins a legal action, the judge will usually award the successful party a. legal aid. b. total costs of the litigation. c. solicitor-client costs. d. party and party costs. e. out-of-pocket expenses only.

d Diff: 1 Type: MC Topic: Procedural Law: Using the Courts Skill: Recall 18

Jack tells Mike, "I'd like to buy your boat for $100 000." Mike, in response, says, "That's too cheap. I'll sell you my boat for $150 000." Mike's statement is a ________ and a ________. a. refusal; new offer b. rejection; new offer c. refusal; rejection d. rejection; counter-offer e. rejection; valid offer

d Diff: 1 Type: MC Topic: Rejection and Counter-offer by the Offeree Skill: Applied 15

When communication between parties is by fax, the place where a contract is formed is a. where the offeror resides. b. where the offeree receives the offer. c. where the subject matter is located. d. where the offeror receives the acceptance. e. where the offeree resides.

d Diff: 1 Type: MC Topic: The Elements of Acceptance Skill: Recall 22

Revocation takes effect a. when an offeror contracts with someone other than the offeree. b. when sent. c. when the offeror becomes bankrupt. d. when received. e. none of the above

d Diff: 1 Type: MC Topic: The Lapse and Revocation of an Offer Skill: Recall 21

An offer is a. a gift. b. an agreement to give something to another party. c. a donation. d. a tentative promise made by one party, subject to a condition or containing a request to the other party. e. a gesture made by one party to another to make a donation.

d Diff: 1 Type: MC Topic: The Nature of an Offer Skill: Recall 2

Which of the following is true? An invitation to do business a. is a legally binding promise. b. is a commitment to enter into a contract upon acceptance. c. is a kind of offer. d. is not an offer. e. is an offer.

d Diff: 1 Type: MC Topic: The Nature of an Offer Skill: Recall 3

Which of the following is true? a. The ordinary standard of care for ordinary negligence, is appropriate when judging the work of a professional. b. Complying with normal professional standards is always an adequate defence. c. Causation in professional-client relationships is not based on reliance. d. A professional who exhibits the degree of skill and knowledge that is commensurate with a particular task will be held to have met the required standard even if the advice given is wrong. e. The duty to take reasonable care does not include omissions.

d Diff: 2 Type: MC Topic: Liability of Professionals Skill: Recall/Applied 19

A year ago, James sold a motor bike to Jim, his best friend. At the time that the bike was sold, Jim was a minor. Jim is still making payments to James. Jim will be attaining the age of majority in a few months' time. How should James protect himself? a. Have Jim transfer the bike to a third party. b. Get a witness to witness the next payment that Jim makes. c. Enter into another agreement with Jim. d. Get Jim to ratify the agreement. e. Provide Jim with a statement of account.

d Diff: 2 Type: MC Topic: Minors (or Infants

On January 1, Jack, a minor, orders some computer software programs from a computer company. Jack needs the programs for school. The software programs cost $4000.00. On January 5, just before the computer company sends the software programs out, Jack has a change of heart. So he calls the computer company and tells them that he no longer wants the programs. In this situation, a. Jack cannot cancel the contract, because there has been offer and acceptance. b. Jack cannot cancel the contract, because the programs are necessaries. c. Jack cannot cancel the contract, because it is a beneficial contract of service. d. Jack can cancel the contract, because the software programs are non-necessaries and he has not taken delivery of them. e. Jack can cancel the contract, because minors do not have to pay large sums of money.

d Diff: 2 Type: MC Topic: Minors (or Infants

Use this fact situation to answer the related questions that follow. Harry, who owns a home in Toronto, needs a plumber to repair a leaky water pipe, so he calls Marty the plumber. Yesterday it snowed, but Harry did not shovel the snow from his walk and it has now turned to ice. Harry has some salt that he could put on the ice to make it safe, but decides that watching the football game is more important. As Marty arrives at Harry's house, he sees Jonny Star, the local newspaper boy, walk up Harry's walk. Harry told Jonny that he could come over to watch the football game on Harry's brand new HD television. Marty gets out of his truck and begins to go up Harry's walk. Of course, because the walk is covered in ice, both Marty and Jonny slip and fall. Marty breaks his right arm, and Jonny breaks his left ankle. When Harry looks out his window, he sees Marty and Jonny on the ground and notices that although Jonny has rubber winter boots on, Marty is wearing his dress shoes, which have leather soles. In the fact situation above, Marty is a. an occupier. b. a trespasser. c. a contractual entrant. d. an invitee. e. a licensee.

d Diff: 2 Type: MC Topic: Occupier's Liability Skill: Applied 10

In order for a person under mental incapacity to avoid a contract made while that person was incapacitated, the incapacitated person must show a. that neither party knew what was happening. b. that there was no intention to enter into a contract. c. that the contract was unfair. d. that the other party should have recognized the incapacity. e. all of the above

d Diff: 2 Type: MC Topic: Other Persons of Diminished Contractual Capacity Skill: Recall 16

When two or more parties have a legal dispute, often the cheapest form of resolving the dispute is by a. litigation in the court. b. mediation. c. arbitration. d. an out of court settlement. e. none of the above

d Diff: 2 Type: MC Topic: Procedural Law: Using the Courts Skill: Applied 13

The Canadian Charter of Rights and Freedoms is said to be entrenched in the Canadian constitution. This means that a. it is not part of the Human Rights Act. b. it is a rule of the Canadian constitution that must be considered by judges only. c. the rights set out in the Charter are absolute. d. it cannot be repealed by an ordinary act of Parliament or of the provincial legislatures. e. judges are given the authority to resolve Charter problems.

d Diff: 2 Type: MC Topic: The Charter of Rights and Freedoms Skill: Recall 26

Which of the following is NOT part of the role or function of the courts? a. to interpret legislation b. to resolve disputes between parties c. to protect civil liberties d. to arrest offenders e. to determine the validity of legislation

d Diff: 2 Type: MC Topic: The Courts and Legislation Skill: Recall 30

Allen is Harry's older brother. Yesterday, Allen asked Harry, an experienced plumber, to fix a leaky pipe below the sink of Allen's main floor washroom. Knowing that Allan had just lost his job and wanting to help him out, Harry did the work, which took him almost half a day to complete. The next day, Harry and Allen had an argument, and in anger Harry told Allen he expected to be paid for his work. In this situation, a. the law will imply a promise by Allen to pay Harry because it deals with supply of services. b. the law will imply a promise by Allen to pay Harry because Allen has been unjustly enriched. c. the law will imply a promise by Allen to pay Harry because they are family members. d. the law will not imply a promise by Allen to pay Harry because they are family members. e. the law will imply a promise by Allen to pay Harry because Harry performed services for Allen.

d Diff: 2 Type: MC Topic: The Effect of a Request for Goods or Services Skill: Recall/Applied 29

Jack is speaking to Michael and Michael happens to mention that he will be painting his home soon. Jack, who is in some financial difficulty, decides to paint Michael's home in the hope that Michael will pay for the service. One weekend, when Michael is away, Jack paints the whole house. He believes that he is entitled to approximately $1500. When Michael returns, Jack approaches him and asks for $1500. Which of the following is true? a. Jack, by painting the home, makes an offer that Michael is free to accept or reject. b. When Michael mentions the need to paint his home to Jack, he indirectly makes an offer that Jack accepts by painting the home. c. Michael has benefited from the painted home and so he is obliged to pay for it. d. Michael is not obliged to pay for the work done because he was never made any offer. e. Jack's conduct is an invitation to do business that has not been accepted by Michael and so Michael is not obliged to pay him.

d Diff: 2 Type: MC Topic: The Elements of Acceptance Skill: Applied 11

Mary places a notice in a local variety store. The notice reads as follows: "I shall pay $1000 for the return of my dog, Mundo." Jim sees the notice. Jim finds the dog and returns it to her. Which of the following is true? a. Mary can refuse to pay Jim because Jim did not communicate his acceptance of the offer to Mary, and an acceptance must be communicated to the offeree. b. The notice is an invitation to treat and could not give rise to a contract. c. Mary makes a counteroffer. d. Mary is obliged to pay Jim because by finding the dog, he accepted the contract and no notification was needed. e. When Jim finds the dog, he makes the offer and it is up to Mary to determine whether she wishes to accept the offer. Mary is thus entitled to reject the offer.

d Diff: 2 Type: MC Topic: The Elements of Acceptance Skill: Applied 28

The contractual requirement of consideration provides that a. the contract must be given careful consideration before it is entered into. b. the offeror must pay cash for the offeree's promise to perform an act. c. an offeree must pay cash for the offeror's promise to perform an act. d. each party must give something of value in exchange. e. what is exchanged by the parties must be of equal value.

d Diff: 2 Type: MC Topic: The Meaning of Consideration Skill: Recall 2

Of the two main classes of legislation, a. active legislation gives the government itself the power to carry on a program and to change the law. b. active legislation gives the government itself the power to change the law. c. passive legislation gives the government itself the power to carry on a program. d. active legislation gives the government itself the power to carry on a program and passive legislation allows it to change the law. e. passive legislation gives the government itself the power to carry on a program and to change the law.

d Diff: 2 Type: MC Topic: The Sources of Law Skill: Recall 10

Specific performance and contempt of court are examples of a. common law rules. b. rules of the law merchant. c. rules of canon law. d. equitable remedies. e. subordinate legislation.

d Diff: 2 Type: MC Topic: The Sources of Law Skill: Recall 23

At what stage in a negligence action will the court apply the remoteness test? a. the duty of care stage b. the breach of duty stage c. the causation stage d. the injury/damages stage e. the standard of care stage

d Diff: 2 Type: MC Topic: Unintentional Torts Skill: Recall 28

Any provision found by a court to be outside of the legislature's jurisdiction and therefore beyond its powers is referred to as a. residual powers. b. concurrent powers. c. rule of law. d. ultra vires.

d Diff: 3 Type: MC Topic: Federalism in the Constitution Skill: Recall 32

Use the fact situation in Q10 to answer the related question that follows. The principle of law that arises out of the fact situation above was meant to overcome an earlier principle of law that stated that a. payment of a lesser sum terminates the contract. b. a contract does not need consideration at all. c. a contract is terminated by performance. d. payment of a lesser sum in satisfaction of a greater sum is no payment at all. e. none of the above

d Diff: 3 Type: MC Topic: Gratuitous Reduction of a Debt Skill: Recall/Applied 14

Which of the following is NOT a defence to a defamation action? a. responsible communication on matters of pubic interest b. the written or spoken words were true c. qualified privilege d. the written or spoken words came from someone else first e. absolute privilege

d Diff: 3 Type: MC Topic: Intentional Torts Skill: Recall 36

In a lawsuit against a professional for a breach of duty of care, a prudent plaintiff should a. sue in contract for breach of contract for failure to properly perform services. b. sue in tort for breach of fiduciary duty. c. sue in tort for negligence arising from a breach of duty of care. d. sue in tort for negligence arising from a breach of duty of care and in contract for breach of contract for failure to properly perform services. e. sue in contract for negligence arising from a breach of duty of care and sue for breach of fiduciary duty.

d Diff: 3 Type: MC Topic: Liability of Professionals Skill: Recall/Applied 5

Janet recently learned that her credit card company charged her an unauthorized amount of 94 cents on her credit card bill. She reviewed the matter and determined that this charge had been made on a monthly basis for several months. She consulted with a lawyer who told her that in the circumstances the best procedural approach to addressing the problem was for her to a. issue a claim in Small Claims Court for the few dollars that she had been improperly charged. b. commence court proceedings in Superior Court seeking a greater amount for legal costs. c. do nothing given the small amount at stake and the costs of litigation. d. seek to have the action against the credit card company certified as a class action thereby allowing her to represent the entire group of possible class members, i.e. those who likewise were billed an unauthorized amount by the credit card company. e. forget any court proceedings but try to attract media attention.

d Diff: 3 Type: MC Topic: Procedural Law: Using the Courts Skill: Applied 17

Which of the following best demonstrates the concept of vicarious liability? a. Jack is employed by Jim as a trainee mechanic. Jack goes to a party and is asked by a friend, Mary, to check her car that won't start. Jack checks the car. A few minutes after Mary drives away, the car's ignition explodes, injuring Maggie. b. A man's dog runs out of the house and mauls a letter carrier. c. A cow leaves its pasture, walks into the garden of another farmer and destroys all the farmer's crops. d. Jack is employed by Jim as a trainee mechanic. Jim's sister, Maggie brings her car to the shop. It has an engine problem. Jack has never worked on a car's engine before. Jim asks Jack to work on the engine. Jack works on the engine. Later when Maggie drives off in the car, the engine explodes, injuring Maggie. e. none of the above.

d Diff: 3 Type: MC Topic: The Basis for Liability Skill: Applied 26

An acceptance of an offer must be a. made slowly. b. loud and clear. c. by conduct only. d. positive. e. in writing only.

d Diff: 3 Type: MC Topic: The Elements of Acceptance Skill: Applied 16

Shahid orders a pizza by telephone and says he will pay on delivery. Which of the following would constitute consideration received by Shahid? a. placing the call. b. promising to pay for the pizza. c. handing over the money. d. accepting delivery of the pizza. e. none of the above.

d Diff: 3 Type: MC Topic: The Meaning of Consideration Skill: Applied 6

The fact that laws that appear to be unjust must still be obeyed suggests that a. justice is merely the result of the influence of ethics. b. justice is merely a product of social morals. c. there is a difference between good laws and bad ones. d. in reality, we must think of justice as nothing more than the result of our legal system. e. those who make the laws just do not care.

d Diff: 3 Type: MC Topic: The Role of Law Skill: Applied 4

Both a crime and a tort are being committed when a. a homeowner decides not to put salt on his walkway after an ice storm and a guest slips and falls the following morning. b. a bar patron becomes ill as a result of being over-served in a tavern. c. an NHL hockey player suffers a broken nose during a consensual on-ice fight. d. a last-minute holiday shopper strikes a fellow shopper with his fist resulting in a broken jaw.

d Diff: 3 Type: MC Topic: The Scope of Tort Law Skill: Applied 22

.

d Diff: 3 Type: MC Topic: The System of Courts in Canada Skill: Applied 11

Use the fact situation in Q6 to answer the related question that follows. In this situation, the preparation of the financial statement by Mr. Wallice, the accountant, constituted a. negligent misrepresentation only. b. breach of contract only. c. negligence only. d. negligence, breach of contract and negligent misrepresentation. e. only negligence and breach of contract.

d Diff: 3 Type: MC Topic: Tort Liability for Inaccurate Statements Skill: Applied 8

A doctor makes a serious error during the course of diagnosing a patient. The patient subsequently dies. The deceased patient's family members bring a lawsuit against the doctor. All the experts who testify agree that the doctor failed to meet the standard of care but that if appropriate care had been provided, the patient would have died in any event. The family members' case against the doctor will a. succeed because all the experts agree the doctor provided substandard care. b. fail because the doctor likely provided the patient with informed consent. c. succeed because the law imposes strict liability upon doctors in similar circumstances. d. fail because the plaintiff cannot prove causation which is an essential element of the tort of negligence.

d Diff: 3 Type: MC Topic: Unintentional Torts Skill: Applied 2

Legislation is the name given to a. laws that are made by judges. b. laws that are made by members of the government. c. the general system of laws of a country. d. laws that are made by Parliament and provincial legislatures. e. none of the above

d Diff: 3 Type: MC Topic: Who Makes Law? Skill: Recall 20

In addition to the two basic categories of public and private law, law is divided further into two more categories, which are a. criminal and contract law. b. domestic and international law. c. criminal and tort law. d. public and private law. e. substantive and procedural law.

e Diff: 1 Type: MC Topic: Classifying Law Skill: Recall 2

Which of the following will a business use to reduce its liability? a. risk reduction b. risk absorption c. risk avoidance d. risk transfer e. all of the above

e Diff: 1 Type: MC Topic: Law and Business Skill: Recall 11

The duties of professionals may be categorized as a. fiduciary, duty in tort, and duty to employees. b. contractual, duty to employees, and duty in tort. c. duty in tort, business duty, and contractual duty. d. general duty, contractual duty, and duty in tort. e. contractual, fiduciary and duty in tort.

e Diff: 1 Type: MC Topic: Liability of Professionals Skill: Recall 2

A minor is a. a person who is small in stature. b. a person who enters into an agreement to work in a coal mine. c. a person who performs minor tasks pursuant to a contract. d. a person who is small in stature but is able to enter into certain contract. e. a person who has not attained the age of majority.

e Diff: 1 Type: MC Topic: Minors (or Infants

Tort damages are often classified into a. special and exemplary damages. b. punitive and exemplary damages. c. special and aggravated damages. d. punitive and special damages. e. general and special damages.

e Diff: 1 Type: MC Topic: Remedies Skill: Recall 3

Use this fact situation to answer the related questions that follow: Alexander enters a grocery store to purchase some tea. He walks through the aisles until he finds the aisle he wants with the different kinds of tea displayed on the shelves. He finds the tea that he wants and then goes to the cashier, who scans the tea and tells Alexander the price, which Alexander then pays. The cashier then hands the tea to Alexander, who leaves the store. In this situation, the tea on the store shelves is a. an offer. b. a contract. c. a promise to sell. d. an invitation to treat. e. all of the above

e Diff: 1 Type: MC Topic: The Nature of an Offer Skill: Applied 4

A tort is a. a criminal act. b. a misrepresentation. c. a breach of contract. d. the punishment imposed on a person for a wrongful act done to the person or property of another. e. a wrongful act done to the person or property of another.

e Diff: 1 Type: MC Topic: The Scope of Tort Law Skill: Recall 21

Use the fact situation in Q6 to answer the related question that follows. In this situation, if the value of the company increases substantially so that Carol makes money on her investment, a. Carol will still suffer damages, but will only have an action against the accountant. b. Carol will still suffer damages, but will only have an action against Henry. c. Carol will still suffer damages, but will only have an action against the bank. d. Carol will still suffer damages and have an action against the accountant and the bank. e. Carol has suffered no damages and has no action at all.

e Diff: 1 Type: MC Topic: Tort Liability for Inaccurate Statements Skill: Applied 11

Where is a contract said to be formed? a. at the place where the offer is made b. at the place where the acceptance is dropped into a mailbox c. at the place where the offer is revoked d. at the place where the parties do business e. at the place when the acceptance becomes effective

e Diff: 1 Type: MC Topic: Transactions Between Parties at a Distance from Each Other Skill: Recall 23

A offers in writing to sell his truck to B for $8000.00. The offer goes on to state that acceptance must be hand-delivered in writing to A. However, B decides to send A a fax in which he accepts A's offer. In this case, a. A must sell his truck to B. b. as far as B is concerned, the only important part of A's offer is B's acceptance in any form. c. the failure of B to accept the offer by fax causes A's offer to lapse. d. B's fax is a proper acceptance of A's offer. e. B's fax is not a proper acceptance of A's offer.

e Diff: 1 Type: MC Topic: Transactions Between Parties at a Distance from Each Other Skill: Recall 25

A valid argument as to why the rule stare decisis should not be an absolute rule is that a. no two sets of facts are identical in every respect. b. fairness in law often requires that judges exercise some flexibility in their approaches to some cases. c. precedents only bind the same set of circumstances. d. judges are only bound to follow decisions of higher courts. e. all of the above

e Diff: 2 Type: MC Topic: Common Law: The Theory of Precedent Skill: Recall/Applied 7

The concept of estoppel is where a. the court stops a party from performing a contract. b. the terms of the contract require that one of the parties be stopped from performing a particular act. c. the promisee is stopped from denying the truth of a fact or statement. d. the court suspends the operation of the contract. e. the maker of a statement or promise is prevented by the courts from either denying the truth of the statement or going back on a promise.

e Diff: 2 Type: MC Topic: Equitable Estoppel Skill: Applied 11

A promise that is made without bargaining for or accepting anything in return is a a. promissory estoppel. b. covenant. c. quantum meruit. d. consideration. e. gratuitous promise.

e Diff: 2 Type: MC Topic: Gratuitous Promises Skill: Applied 24

Use this fact situation to answer the related questions that follow. On June 1, A, a debtor, loans $5000.00 from B, a creditor, and agrees to repay it by December 1 of the same year. On November 1, B says to A, "Rather than paying me the full $5000.00, just pay me $3500.00 and your debt will be cancelled." A pays the $3500.00, but B now changes his mind and tells A that he wants A to pay him the balance of $1500.00. In this situation at common law, a. B cannot recover the extra $1500.00, because he agreed to reduce the amount of the debt. b. B is estopped by promissory estoppel from recovering the $1500.00. c. B can recover the extra $1500.00 on the basis of quantum meruit. d. B can recover the extra $1500.00 under present-day legislation. e. B can recover the extra $1500.00 since his promise lacks fresh consideration.

e Diff: 2 Type: MC Topic: Gratuitous Reduction of a Debt Skill: Recall/Applied 27

Michael is employed by James' Groceries. Jim, the owner of Jim's Groceries approaches Michael and convinces Michael to leave James' Groceries and work for Jim's Groceries. James' Groceries may sue Jim's Groceries for a. breach of employer-employee contract. b. breach of employee contract. c. interference with employee affairs. d. breach of contract. e. inducing breach of contract.

e Diff: 2 Type: MC Topic: Intentional Torts Skill: Applied 15

A risk management plan a. does not require regular revision. b. does nothing to promote the better conduct of a business. c. is only necessary in sophisticated businesses. d. does not necessarily involve a lawyer in every part of the overall plan. e. helps a business to identify and anticipate potential legal liability.

e Diff: 2 Type: MC Topic: Law and Business Skill: Applied 10

When we say that the law is part of every facet of business, we mean that a. it is central to business interaction with customers, suppliers, competitors, and government. b. business cannot succeed without understanding the law. c. it outlines what we do, how to do it, and what not to do. d. the failure of businesses to comply with the law may result in criminal, regulatory, and/or civil liability. e. all of the above

e Diff: 2 Type: MC Topic: Law and Business Skill: Recall 8

Effective operation of a business requires that business managers a. rely on lawyers to form the right questions and answers for legal issues. b. do not have to become experts in the activities of their businesses. c. simply rely on the decisions made by those in authority above them. d. do not have to familiarize themselves with the law. e. must know the law and must understand the legal risks associated with the everyday choices that they make.

e Diff: 2 Type: MC Topic: Law and Business Skill: Recall 9

A lawyer who enters into a business transaction with a client and neglects to inform the client that his credit rating is poor may be found to be a. in breach of the law that governs lawyers. b. in breach of his or her duty to inform the client. c. in breach of contract. d. in breach of the common law. e. in breach of his or her fiduciary duty.

e Diff: 2 Type: MC Topic: Liability of Professionals Skill: Recall 17

Use this fact situation to answer the related questions that follow. On January 1, A and B enter into a contract under which B agrees to renovate A's house by September 1 for $60 000.00. By June 2, B has not yet started the renovations and A is getting worried, so A tells B that he will pay him an extra $5000.00 if B completes the renovations on time. B completes the renovations by September 1st and now asks A for $65 000.00. In this situation, a. A's promise to pay B an extra $5000.00 is not supported by fresh consideration. b. A's promise to pay B an extra $5000.00 is made before the contract is over. c. the consideration for the contract on January 1 is $60 000.00. d. there was a contract on January 1 that must be performed by September 1. e. all of the above

e Diff: 2 Type: MC Topic: Motive Contrasted With Consideration: Past Consideration Skill: Applied 25

When considering whether or not it is worthwhile to commence an action or lawsuit , a prudent business manager must consider a. the staggering costs of the litigation, even if winning appears to be a certainty. b. that even if winning appears to be a certainty, it may be impossible to collect any money from the defendant. c. that there is always a risk of losing and having to pay the legal costs of the other party. d. that even if winning appears to be a certainty, the litigation may take up a huge amount of the business manager's time, which would be better spent concentrating on the business. e. all of the above

e Diff: 2 Type: MC Topic: Procedural Law: Using the Courts Skill: Applied 15

In class action proceedings, a. legal aid is available to injured plaintiffs. b. the court appoints a lawyer to help injured parties. c. a court usually awards punitive damages against negligent manufacturers. d. separate claims are consolidated for trial. e. a plaintiff applies to court to represent a class of plaintiffs with similar claims.

e Diff: 2 Type: MC Topic: Procedural Law: Using the Courts Skill: Recall 26

A professional is a. a person who is licensed by various levels of government. b. a person who has a special accreditation or degree from a professional organization. c. a person who belongs to a special club. d. someone who sets him- or herself out to help the public. e. someone who has specialized knowledge and skill on which a client relies and for which a client is prepared to pay.

e Diff: 2 Type: MC Topic: Professional Liability: The Legal Challenges Skill: Applied 24

John and Edward are negotiating the sale and purchase of one of John's very expensive paintings. In the course of the negotiations, John says, "I'll sell it to you for $20 000.00, to which Edward replies, "That is a little too much for me; I'll give you $14 500.00." John thinks about this for a moment and says, "No, but I'll take $17 500.00. Edward says, "That is still too much; I'll buy it for $15 500.00. John looks at the painting and replies, "You know it is worth more than that. Why don't you give me $16 500.00 and we'll call it a day." Edward responds with, "I only have $16 000.00 so that is all I will pay for it," to which John replies, "Done." In this case, a. the contract is for $16 500.00. b. the contract is for $17 500.00. c. the contract is for $14 500.00. d. there is no contract at all. e. the contract is for $16 000.00.

e Diff: 2 Type: MC Topic: Rejection and Counter-offer by the Offeree Skill: Applied 27

In modern times, the basis for liability in tort is a. whether the wrongdoer is able to compensate the victim. b. actionable recklessness. c. legal carelessness. d. criminal wrongdoing. e. fault.

e Diff: 2 Type: MC Topic: The Basis for Liability Skill: Recall 24

In deciding whether a statute is valid or invalid, the court must consider a. the effect that the statute has. b. the purpose of the statute. c. whether it should interpret the statute broadly. d. whether it should interpret the statute narrowly. e. all of the above

e Diff: 2 Type: MC Topic: The Courts and Legislation Skill: Recall 13

Joe finds a dog wandering in his yard and he lets it into the house. A day later he sees an advert on a tree stating that the owner of the dog will pay $100 to anyone who finds and returns the dog to the local SPCA office. Joe takes the dog to the local SPCA office. Which of the following is true? a. Joe did not communicate his acceptance of the offer to the SPCA and so no contract arose. b. The kind of advert that is placed on the tree cannot give rise to a contract. c. When Joe takes the dog to the SPCA, he makes the offer and if the SPCA accepts the offer, they are obliged to pay him. d. The advert on the tree is an invitation to do business and cannot give rise to a contract. e. Joe will be entitled to the reward because his performance of the obligation in response to the offer is all that is needed in a unilateral contract.

e Diff: 2 Type: MC Topic: The Elements of Acceptance Skill: Applied 10

Margaret finds a stray cat and takes it in. The following day she is speaking to some friends and learns that Kelly, a woman who lives some distance away, has lost a cat. She goes around to the home of Kelly, leaves the cat in the yard and leaves a note for Kelly. The next day she gets a voice mail message from Kelly thanking her. A few days later Margaret sees a notice in which Kelly offers to pay $500 to anyone who returns her cat. Margaret, who needs some cash, returns to Kelly's home and demands the $500 reward. Which of the following is true? a. Kelly's notice is an invitation to treat. b. Because Margaret did not demand the $500 when she returned the cat, she cannot go back and request it. The offer has lapsed. c. By returning the cat, Margaret performed the contract and so has to be paid. Margaret's performance gives rise to a unilateral contract. d. Kelly and Margaret have a bilateral contract. e. Margaret did not know of the offer before she returned the cat. So she could not have been accepting the offer when she returned the cat. No contract arose. There is thus no obligation on Kelly to pay the $500.

e Diff: 2 Type: MC Topic: The Nature of an Offer Skill: Applied 29

We need to have law because it a. regulates individuals' interactions with one another. b. gives the government the power to act for the benefit of society in general. c. provides an element of certainty in determining contractual and property rights. d. protects persons, property, and society, and prohibits conduct that society believes is harmful. e. all of the above

e Diff: 2 Type: MC Topic: The Role of Law Skill: Recall 3

On March 1, Al agrees to sell Barb his house for $250 000. Barb pays a deposit of $15 000, and they agree that title will transfer and the price will be paid on April 30. The agreement is signed by both parties under seal. What is the consideration on Barb's part? a. the deposit of $15 000 b. there is no consideration c. quantum meruit d. the seal e. the promise to pay the $250 000

e Diff: 2 Type: MC Topic: The Use of a Seal Skill: Applied 7

Negligent misrepresentation is a. a defamatory statement. b. a statement that is fraudulent. c. a statement that is made by one party to another and induces the other person to engage in negligent acts. d. a statement that insults another. e. an incorrect statement that is made without due care for its accuracy.

e Diff: 2 Type: MC Topic: Tort Liability for Inaccurate Statements Skill: Recall 15

Mary, a prostitute, enters into a contract with a man to provide sexual services in return for an agreed-on amount of money. In this situation, a. no remedy is available to Mary if the man does not pay her. b. no remedy is available to the man if he pays the money and Mary refuses to provide her services. c. the contract is illegal and therefore unenforceable. d. there is a contract between Mary and the man. e. all of the above

e Diff: 3 Type: MC Topic: Contracts Illegal by the Common Law and Public Policy Skill: Applied 28

Use this fact situation to answer the questions that follow it. In January 1941 in Canada, John Landlord and Harry Tenant enter into a lease under which Harry agrees to rent a house from John for five years at a rent of $1000.00 per month. In March 1941, Canada declares war on Germany and John tells Harry that until the war is over, Harry will not have to pay rent. Of course, Harry stops paying rent while the war goes on. The war ends in January 1945, and now John Landlord sues Harry for all of the rent from March 1941 to present. In turn, Harry is thinking of suing John Landlord on the basis that his statement to Harry about not paying the rent during the war was a contract. In the fact situation above, a. John Landlord can argue that he is entitled to the rent, because there was no consideration for his promise to Harry, and the court will accept this. b. Harry will be able to successfully sue John Landlord for breach of his agreement not to make Harry pay rent during the war. c. John Landlord can argue that it would be unjust for Harry not to have to pay the rent, and this argument will succeed. d. at the very least the court will value John Landlord's claim on a quantum meruit basis. e. Harry Tenant will argue that John Landlord's promise in March 1941 stops John Landlord from suing him for rent from March 1941 to the end of the war, and this argument will be a good defence to John Landlord's action.

e Diff: 3 Type: MC Topic: Equitable Estoppel Skill: Applied 12

Use the fact situation in Q10 to answer the related question that follows. The principle of contract law that arises in the fact situation above is a. revocation. b. fraudulent misrepresentation. c. deceit. d. fresh consideration. e. promissory estoppel.

e Diff: 3 Type: MC Topic: Equitable Estoppel Skill: Applied 13

To implement a risk management plan, a business must first a. prioritize the risks. b. develop effective strategies to deal with the risks. c. revise the plan on a regular basis. d. implement the plan. e. conduct a legal audit of the business operation.

e Diff: 3 Type: MC Topic: Law and Business Skill: Applied 12

When we talk about predictability being a major element of law, we mean that a. although the law is predictable, the outcome of a given dispute between people is not. b. people should be more likely to be able to predict when they can circumvent a law. c. the law itself is not predictable, only the cases that are decided. d. by using common sense, the decision of any court can be determined in advance. e. people should be able to find out where they stand and how to act with reasonable certainty.

e Diff: 3 Type: MC Topic: Legal Systems: Civil Law and Common Law Skill: Recall/Applied 5

. Henry, needing a loan from his bank, went to his accountant, Bob Wallice, and asked him to prepare a financial statement of the company that Henry could give to the bank to get the loan. Unfortunately, Mr. Wallice included certain property that did not belong to the company, which increased its value. When Henry took the financial statement to the bank, the bank caught the error, but made Henry guarantee the loan. The bank then told Mr. Wallice, who forgot to correct the error in the financial statement. A few days later, Henry decided he needed to invest more money in his company and began negotiations with a business associate, Carol Hill. When Carol asked for the financial statement of the company, Henry told her she could get it from the bank or from his accountant. Carol decided to go to the bank, which gave her the financial statement with the error in it. Based on the financial statement, Carol invested $50 000.00 in the company, only to realize a month later the problem with the financial statement the bank had given to her. In this situation, a. Mr. Wallace owed a fiduciary duty to Carol. b. the bank owed a fiduciary duty to Carol. c. Mr. Wallace owed a fiduciary duty to Harry. d. Mr. Wallace owed a fiduciary duty to the bank. e. all of the above

e Diff: 3 Type: MC Topic: Liability of Professionals Skill: Applied 7

A conflict of interest can be said to arise a. where a professional represents two or more parties, each of whom has an opposing interest. b. when a client's interest conflicts with the interest of another person represented by the same professional. c. where a client's interest conflicts with the interest of the professional representing the client. d. where a professional represents two parties, each of whom has a competing interest. e. all of the above

e Diff: 3 Type: MC Topic: Liability of Professionals Skill: Recall/Applied 4

Standing to sue is recognized by a court when a litigant a. pays money into court to the credit of the action he or she has brought. b. enters an appearance by filing a notice of intention to contest an action. c. files an affidavit containing hearsay evidence. d. fails to defend an action. e. has a direct interest in a matter and/or whose rights are specifically affected by another.

e Diff: 3 Type: MC Topic: Procedural Law: Using the Courts Skill: Recall/Applied 31

Use the fact situation in Q3 to answer the related question that follows. If Mary sues the university, among the damages that she will receive if she is able to prove her case are a. general and special damages for pain and suffering and for her broken leg and sprained arm. b. general damages for loss of income and medical expenses, etc. c. special damages for pain and suffering and for her broken leg and sprained arm. d. general and special damages for loss of income and medical expenses, etc. e. general damages for pain and suffering and for her broken leg and sprained arm and special damages for loss of income and medical expenses, etc.

e Diff: 3 Type: MC Topic: Remedies Skill: Recall/Applied 8

Strict liability a. absolves the defendant from all liability. b. establishes the amount of damages that is payable to an injured party. c. requires that the plaintiff strictly prove that the defendant was at fault. d. means that liabilities are strictly enforced. e. removes the onus on the plaintiff to show that the defendant is at fault.

e Diff: 3 Type: MC Topic: The Basis for Liability Skill: Applied 25

The "notwithstanding" provision in the Charter of Rights and Freedoms is one that a. allows a court to decide that notwithstanding the fundamental freedoms specified in the Charter, the judgment of the court is supreme. b. allows a legislature to pass laws that override the fundamental freedoms specified in the Charter. c. allows a legislature to pass a law that provides that notwithstanding the law that has been passed the Charter of Rights and Freedoms will still apply. d. allows a legislature to pass a law notwithstanding a court order that prohibits it from doing so. e. allows a legislature to pass laws that override the fundamental freedoms specified in the Charter if the provision specifically states that it shall operate notwithstanding the freedoms specified in the Charter.

e Diff: 3 Type: MC Topic: The Charter of Rights and Freedoms Skill: Applied 29

property to B for $6000.00. The offer stated that it would be open for acceptance until 9 a.m. on June 12. On June 11, B heard that A was agreeing to sell or had sold his property to C. On the evening of June 11, B delivered an acceptance of A's offer to the place where A was staying and on the following morning personally delivered his acceptance to A, who declined the acceptance stating, "You are too late." In this situation a. A's offer must remain open until 9 a.m. on June 12th. b. B's acceptance of A's offer is a valid acceptance. c. because B did not give A the money, there is no acceptance. d. B's failure to accept A's offer immediately constitutes a rejection of A's offer. e. having heard that A had sold or was selling the property to C, B knows A's offer has been revoked.

e Diff: 3 Type: MC Topic: The Lapse and Revocation of an Offer Skill: Applied 14

When we say that the law is linked to moral and ethical standards, we mean that a. the law is based on ethics. b. ethical behaviour is generally considered to be a higher standard. c. ethics and morality are one and the same. d. the fundamental truths that give rise to the law include ethics and morality. e. the moral and ethical values of a society as a whole shape the development of the law.

e Diff: 3 Type: MC Topic: The Role of Law Skill: Applied 16

Use the fact situation in Q6 to answer the related question that follows. In this situation Henry's telling Carol to get the financial statement from the bank constitutes a. a negligent misrepresentation. b. an innocent misrepresentation. c. negligence. d. a breach of the duty to act in good faith. e. none of the above

e Diff: 3 Type: MC Topic: Tort Liability for Inaccurate Statements Skill: Applied 10

Today, the rule in Foakes v. Beer, that payment of a lesser sum in satisfaction of a greater sum is not payment at all, does not apply in Ontario. Explain.

Ontario, together with a number of other provinces, has passed legislation that overcomes the rule in Foakes v. Beer to give effect to agreements called settlements, which settle a debt. Diff: 2 Type: ES Topic: Gratuitous Reduction of a Debt Skill: Recall 45

When we say that consideration is the price for a promise, what does price mean?

Price is the corresponding obligation of the other party for the promise made to him or her or it and can take the form of money payment, the performance of some act, or the conscious decision not to perform an act. Diff: 1 Type: ES Topic: The Meaning of Consideration Skill: Recall 43

What are professional bodies? Explain the role of professional bodies.

Professional organizations are autonomous organizations established under provincial statutes for the purposes of determining and policing standards of compliance and behaviour among members of the profession. Professional organizations will usually have governing bodies that set rules and regulations for members. Some of the responsibilities assumed by professional bodies include (a

Professionals may have liability to their clients in tort and contract, and for breach of fiduciary duty. Briefly explain the basis for liability in these categories.

Professionals may be liable for torts committed against clients and for breach of contract. In general, any agreement to perform services for a client contains an agreement that the professional will perform the service with due care. Where the professional does not perform the service with due care, there is breach of contract and the client may sue for damages. In addition to this, a breach of contract will also occur where the professional fails to perform the service, or performs unsatisfactorily. Clients will also have a claim in tort under a variety of circumstances. These include negligence and negligent misstatements. A professional person also has a fiduciary duty. One of the consequences of this obligation is the duty of the fiduciary or professional not to place herself or himself in a position that will give rise to a conflict of interest—that is, a position where his or her interest will conflict with his or her duty. Thus, a professional will usually not be expected to act on behalf of two persons who have competing interests because the professional will find it hard to fulfill his or her duty to both parties. Diff: 3 Type: ES Topic: Liability of Professionals Skill: Applied 55

Explain what is meant by the "duty to account" of a professional who has committed a breach of trust.

Professionals or other persons in fiduciary relationships are prohibited from making secret profits as a result of breaching their fiduciary duty. Where such a professional or person makes a secret profit or profits from her or his position as a fiduciary, she or he will owe a duty to account or hand over all the profits she or he made as a result of the breach. Diff: 2 Type: ES Topic: Liability of Professionals Skill: Recall/Applied 49

The Charter of Rights and Freedoms provides that all are entitled to certain fundamental freedoms. What are these fundamental freedoms? Are there any limits on these freedoms?

Section 2 of the Charter of Rights and Freedoms provides that everyone has the following freedoms: (a

Explain why it is important that the Supreme Court of Canada not be bound by its own decisions.

The Supreme Court of Canada is the highest court in the country and the court of last resort for all matters. If it were to bind itself to it own decisions, it would make it virtually impossible for the law to change. As the highest court in Canada, the Supreme Court of Canada must have the flexibility to change the law when contemporary standards change. For this reason it must be able to review and reverse its own earlier decisions, thereby keeping the law current and up to date. Diff: 3 Type: ES Topic: Common Law: The Theory of Precedent Skill: Applied 58

Injurious reliance and equitable estoppel are two sides of the same coin. Explain.

The U.S. principle of injurious reliance and the English principle of equitable estoppel are two sides of the same coin. The concept of injurious reliance examines the loss that has been sustained by a promisee who has relied on the gratuitous promise to his or her detriment. Under the English principle of equitable estoppel, the court examines matters from the position of the promisor. It determines whether the promisor made a promise that it would be unfair for it to renege on, and then estops the promisor from doing so. Diff: 2 Type: ES Topic: Equitable Estoppel Skill: Recall/Applied 51

Explain the theory behind lapse as a way of preventing the formation of a contract.

The common law requires that the making of a contract should take place in a timely fashion. For this reason it was considered unrealistic that offers should remain open for acceptance forever or for inordinate amounts of time. This view was also applied in situations where the very nature of the subject matter of the offer (such as fruit or some other perishable

You are a judge sitting on the Supreme Court of Canada, which has been asked to rule on the interpretation of a provision in a new statute and to decide whether it is constitutional. What are the two approaches that you can take and what would be the outcome of each?

The first approach is to determine whether the words should be interpreted in a broad or restrictive manner. A broad interpretation might result in the provision being found unconstitutional, while a restrictive meaning would leave it valid. The second approach is to try to avoid interpreting the provision so narrowly that the result does not interfere with existing private rights, which may have the effect of frustrating any intended reform. Diff: 3 Type: ES Topic: Challenging the Validity of a Statute Skill: Applied 58

In a lawsuit alleging negligent misrepresentation the plaintiff must establish more than a breach of duty causing loss. Identify the five requirements for proving negligent misrepresentation as set by the Supreme Court of Canada.

The five requirements for proving negligent misrepresentation are: i. There must be a duty of care based on a "special" relationship between the representor and the representee. ii. The representation in question must be untrue, inaccurate, or misleading. iii. The representor must have acted negligently in making a representation—that is, he or she must have fallen below the requisite standard of care required of a professional making such a representation. iv. The representee must have relied, in a reasonable manner, on the negligent misrepresentation. v. The reliance must have been detrimental to the representee in the sense that damages resulted. Diff: 3 Type: ES Topic: Proving Negligent Misrepresentation Skill: Recall/Applied 1

The standard of care in the law of negligence is not a uniform standard. Explain.

The law places a duty on each person to take such reasonable care that his or her acts and omissions do not cause foreseeable harm to another. The standard of care, that is the nature and degree of care that the law considers to be reasonable and expects a person to live up to, is not uniform and varies from person to person and from activity to activity. In addition, the court also takes into consideration various factors, such as the likelihood of harm and the potential severity of the injury. Thus, in the case of a medical procedure with potentially serious consequences, the standard of care that the court will expect of a doctor who is a specialist will be higher than the standard it expects of someone who is not a specialist. Diff: 3 Type: ES Topic: Unintentional Torts Skill: Applied 75

What must a minor do if he or she wishes to avoid liability under a contract?

The minor must promptly repudiate the contract and return goods not used, whatever their condition. Diff: 1 Type: ES Topic: Minors (or Infants

What is res judicata?

The term res judicata means that a matter has already been decided by the courts and cannot be re-argued by the same parties. For instance, if Jack sues a corporation for negligence and the court hands down a judgment, then the matter becomes res judicata and Jack cannot re-argue the case. Diff: 1 Type: ES Topic: Procedural Law: Using the Courts Skill: Recall/Applied 54

Generally speaking, there are three types of codes of conduct that may be imposed on a business. List or state these codes of conduct.

The three types of codes of conduct are binding codes, voluntary codes and self-imposed codes. Diff: 1 Type: ES Topic: Law and Business Ethics Skill: Recall 52

For the most part, Canadian courts apply modified principles of negligence to determine liability. Some critics of fault-based systems propose that victims be compensated through "no-fault" government schemes—similar to those already in place relating to motor vehicle accidents and workers compensation. Others argue that the law should impose "strict liability" in all areas in which plaintiffs have traditionally struggled to establish fault, namely, against manufacturers of goods or products. Explain the three bases for awarding compensation to injured parties and the advantages and disadvantages of each.

The traditional fault-based system provides an injured plaintiff with access to the courts to obtain compensation. However, a plaintiff must prove all three elements of a traditional negligence claim—that being, fault causing loss. Fault-based systems are often thought to be fair because damages are only awarded against negligent defendants. There is also a significant deterrent effect. However, requiring a plaintiff to establish fault can be costly and can often result in a financially needy plaintiff receiving no compensation at all. A comprehensive no-fault scheme with respect to personal injury would in effect take away the right to sue for compensation. Instead, everyone who suffered unexpected or unanticipated injury could apply pursuant to a government scheme to receive compensation. No-fault schemes are attractive in that all injured parties receive some level of compensation, but a government scheme of no-fault insurance for all personal injury would undoubtedly be costly and claimants would likely receive lower levels of compensation than they would under the current tort regime. Critics of no-fault legislation also argue that persons may be less vigilant in causing injury to themselves or in contributing to any loss. Additionally, there would be no deterrent effect on potential defendants to take care in circumstances where parties may be injured as a result of their acts or omissions. To impose strict liability on all defendants would certainly allow plaintiffs to more easily recover damages for their injuries. There would be a requirement on the part of the plaintiff to muster a case on the issue of liability, which often would require hiring lawyers and experts. It is arguable that defendants would take greater steps to ensure that the products they delivered to market were safe for users. On the other hand, imposing liability on every defendant in every case where a plaintiff suffered harm could potentially result in ballooning insurance costs. A strict liability scheme would likely lead to some legislative measures reducing damage awards to avoid runaway insurance premiums. The plaintiff retains access to the courts but is relieved from any obligation to prove fault, rather shifting the obligation to the defendant to show that no breach of the applicable standard of care occurred. Diff: 3 Type: ES Topic: The Basis for Liability Skill: Critical Thinking 1

Causation is the third condition that must be proven to sustain an action for tort liability. Explain.

There are three conditions that must be proven to sustain a finding of tort liability: that the defendant owed a duty to the injured party, that the defendant breached this duty, and that the breach of that duty caused the injury. It is the last of these conditions that causation deals with. In this context, the question to be asked is whether the injured party relied and acted on the advice of the professional to his or her detriment. If the answer is yes, then the three conditions have been met and liability will follow. Diff: 2 Type: ES Topic: Causation Skill: Recall/Applied 45

When one party to a concluded contract promises the other party something more, why is the new promise not binding?

There is no fresh consideration for the new promise. Diff: 2 Type: ES Topic: Gratuitous Promises Skill: Applied 44

A bartender employed in a licensed establishment over-serves a patron. As a result of the over-service, the patron physically assaults another patron by striking him with a beer bottle. Identify and describe the three forms of liability that may arise as a result of this single incident.

There may very well be criminal charges laid against the patron who committed the assault. This is a matter of public law and would impose the most serious consequences as a result of this criminal act. Additionally, the injured patron may very well sue for compensation as a result of the injuries sustained. The court would be required to determine whether the patron who committed the assault and/or the bartender who over-served him are civilly liable. Finally, liquor licence authorities may request a tribunal hearing to determine whether the tavern is responsible for violating regulatory laws that govern the service of alcohol in a licensed establishment. Diff: 3 Type: ES Topic: Forms of Legal Liability Skill: Applied 1

List four kinds of rights that are covered by the Canadian Charter of Rights and Freedoms.

These are equality, mobility, legal, and democratic rights. Diff: 2 Type: ES Topic: The Charter of Rights and Freedoms Skill: Recall 55

Explain the legal system commonly referred to as civil law.

This system of law is one that was derived from Roman law, and more particularly Justinian's Code; it involves a comprehensive legislated code in which the actual laws are codified or written down in a statute. Diff: 2 Type: ES Topic: Legal Systems: Civil Law and Common Law Skill: Recall/Applied 51

Criminal law and tort law focus on different persons when a tort or crime is committed. Explain.

Tort law focuses on the victim of a tortious act and seeks to compensate the victim for the harm suffered. Criminal law focuses on the perpetrator of the criminal act and seeks to punish the perpetrator. Diff: 3 Type: ES Topic: The Basis for Liability Skill: Applied 71

Joe hired Julie, a lawyer, to defend him on assault charges. At their first meeting he told her the details concerning his involvement in the assault. After a dispute between Joe and Julie in relation to legal fees, Joe decided to retain a new lawyer. The police attended at Julie's office to ask her questions concerning what Joe told her about the assault. In these circumstances a. the solicitor/client privilege does not allow Julie to reveal any communications she had with Joe without his approval. b. given that the solicitor/client relationship has ended, Julie is at liberty to disclose the information. c. the solicitor/client privilege does not extend to admissions by clients of wrongdoing so communications must be disclosed. d. Julie is at liberty to disclose any information in relation to the dispute with Joe over fees but not in relation to communications about the assault. e. the Canada Evidence Act permits Julie to disclose any information once the retainer has ended.

a Diff: 3 Type: MC Topic: Legal Risk Management Skill: Applied 6

Why is hearsay evidence generally not admissible in a trial?

Witnesses who testify in court should be subject to cross-examination so the court can assess their credibility. Hearsay evidence is evidence of words attributed by a witness to a person who is not before the court. That evidence should not be allowed because the person who is alleged to have said the words cannot be cross-examined by the opposing lawyer. Diff: 3 Type: ES Topic: Procedural Law: Using the Courts Skill: Applied 56

The contractual duty of a professional contains a promise a. to perform the services with due care. b. to accept payment rendered for services. c. to work for the client. d. to engage other agents in pursuance of the contract. e. to submit invoices denominated in Canadian dollars only.

a Diff: 1 Type: MC Topic: Liability of Professionals Skill: Recall 14

A minor may repudiate all contracts entered into by her with the exception of a. contracts for necessaries. b. contracts entered into under duress. c. contracts that are beneficial to the minor. d. contracts pertaining to real estate. e. contracts entered into with relatives.

a Diff: 1 Type: MC Topic: Minors (or Infants

Native Canadians living on reserves are considered to be ________ of the Crown. a. wards b. children c. beneficiaries d. friends e. relatives

a Diff: 1 Type: MC Topic: Other Persons of Diminished Contractual Capacity Skill: Recall 20

A standing offer is a. one that may be accepted from time to time as needed. b. one that can be revoked even after acceptance. c. an offer that is made and withdrawn before it is accepted. d. one that is said to stand because it is fixed and cannot be changed. e. one that cannot be revoked after acceptance without the consent of the offeree.

a Diff: 1 Type: MC Topic: The Elements of Acceptance Skill: Recall 20

When an offer is not accepted, it will a. lapse. b. no longer be an offer. c. terminate. d. end. e. be void.

a Diff: 1 Type: MC Topic: The Lapse and Revocation of an Offer Skill: Recall 12

A multi-disciplinary partnership is a. a partnership between two or more persons who belong to different professions. b. a partnership between two or more persons who belong to the same profession. c. a partnership of persons in the same profession who have different specializations. d. a limited partnership. e. a partnership whose members have been disciplined for misconduct.

a Diff: 1 Type: MC Topic: The Role of Professional Organizations Skill: Recall 22

The role of tort law is to a. compensate victims for harm suffered from the activities of others. b. assist judges by providing rules to determine how much money to pay victims of crime. c. punish wrongdoers in the same way that criminal law punishes criminals. d. determine who caused a motor vehicle accident. e. impose strict rules to prevent the commission of crimes.

a Diff: 1 Type: MC Topic: The Scope of Tort Law Skill: Recall 20

In order to collect compensation for a loss under a fire insurance policy, the insured must show an insurable interest. a. True Correct: Correct b. False Incorrect: Incorrect

a Diff: 1 Type: TF Topic: Contracts Affected by Statute Skill: Recall 38

Gratuitous promises are made without consideration. a. True Correct: Correct b. False Incorrect: Incorrect

a Diff: 1 Type: TF Topic: Gratuitous Promises Skill: Recall 37

Legal liability is the term used when a person breaks the law. a. True Correct: Correct b. False Incorrect: Incorrect

a Diff: 1 Type: TF Topic: Law and Business Skill: Applied 38

Liability for breach of a fiduciary duty may arise even where there has been no negligence. a. True Correct: Correct b. False Incorrect: Incorrect

a Diff: 1 Type: TF Topic: Liability of Professionals Skill: Recall 36

There are two models of legal aid that are used in Canada. a. True Correct: Correct b. False Incorrect: Incorrect

a Diff: 1 Type: TF Topic: Procedural Law: Using the Courts Skill: Recall 44

Most provinces in Canada permit contingency fee arrangements. a. True Correct: Correct b. False Incorrect: Incorrect

a Diff: 1 Type: TF Topic: Procedural Law: Using the Courts Skill: Recall 45

A fiduciary duty may exist in the absence of a contractual duty. a. True Correct: Correct b. False Incorrect: Incorrect

a Diff: 1 Type: TF Topic: Professional Liability: The Legal Challenges Skill: Recall 30

An innocent employer is vicariously liable for the negligent acts of an employee. a. True Correct: Correct b. False Incorrect: Incorrect

a Diff: 1 Type: TF Topic: Strategies to Manage the Legal Risks Skill: Recall 41

A void contract is one that is deemed in law to have never been created. a. True Correct: Correct b. False Incorrect: Incorrect

a Diff: 1 Type: TF Topic: The Difference Between a Void and an Illegal Contract Skill: Recall 29

Certain fundamental truths, such as "all men are created equal," have been incorporated into our laws. a. True Correct: Correct b. False Incorrect: Incorrect

a Diff: 1 Type: TF Topic: The Role of Law Skill: Recall 35

Generally speaking, all professional organizations have a number of the same responsibilities. a. True Correct: Correct b. False Incorrect: Incorrect

a Diff: 1 Type: TF Topic: The Role of Professional Organizations Skill: Recall 33

Like any other standard of care, the standard of care for professionals evolves and changes as the law evolves and changes. a. True Correct: Correct b. False Incorrect: Incorrect

a Diff: 1 Type: TF Topic: The Standard of Care for Professionals Skill: Recall/Applied 34

A document made under seal does not require consideration. a. True Correct: Correct b. False Incorrect: Incorrect

a Diff: 1 Type: TF Topic: The Use of a Seal Skill: Recall 41

A professional may owe a duty of care in tort to someone other than the client. a. True Correct: Correct b. False Incorrect: Incorrect

a Diff: 1 Type: TF Topic: Tort Liability for Inaccurate Statements Skill: Recall 37

A duty of care may be owed to someone other than the person who is directly injured. a. True Correct: Correct b. False Incorrect: Incorrect

a Diff: 1 Type: TF Topic: Unintentional Torts Skill: Applied 37

An act of omission that is not found to be negligence today may later be found to be negligence. a. True Correct: Correct b. False Incorrect: Incorrect

a Diff: 1 Type: TF Topic: Unintentional Torts Skill: Recall/Applied 40

Substantive law deals with the rights and duties of each person in society, and procedural law deals with the machinery to enforce those rights. a. True Correct: Correct b. False Incorrect: Incorrect

a Diff: 1 Type: TF Topic: Who Makes Law? Skill: Recall 32

Business agreements are most often challenged as contrary to public policy because a. they may be in restraint of trade. b. the agreement may be intended to defraud others. c. one party may be more powerful than the other. d. one party may have been coerced into signing the agreement. e. the agreement may be contrary to statute.

a Diff: 2 Type: MC Topic: Agreements in Restraint of Trade Skill: Recall 22

The principle of injurious reliance is a. the loss or harm that is suffered by a promisee who, to his or her detriment, relies on a gratuitous promise. b. the contractual relationship that arises between an injured party and the person or persons on whom this injured party relies. c. the injury that a person in a contractual relationship suffers when there is a breach of contract. d. the amount of compensation that is awarded to a party where there is a breach of contract. e. the personal injury that is suffered by persons who are involved in contractual situations.

a Diff: 2 Type: MC Topic: Equitable Estoppel Skill: Applied 10

Which of the following is NOT a defence to the tort of defamation? a. that the defendant did not mean to harm the plaintiff's reputation b. that what the defendant said was true c. that the plaintiff's reputation was not damaged d. qualified privilege e. absolute privilege

a Diff: 2 Type: MC Topic: Intentional Torts Skill: Recall 14

Which of the following is NOT true? a. A fiduciary duty exists when there is an existing contract between the professional and the client. b. The principles for determining damages for breach of contract and breach of fiduciary duty are not necessarily the same. c. A fiduciary duty requires that a professional avoid situations that give rise to a conflict of interest. d. The existence of a fiduciary duty means that a lawyer cannot, as a general rule, act for a purchaser and vendor at the same time. e. A fiduciary duty requires complete fidelity and loyalty to the other party.

a Diff: 2 Type: MC Topic: Liability of Professionals Skill: Applied 27

A minor's immunity from liability a. is limited to contracts. b. is determined by statute. c. is dependent on the age of the minor. d. extends to contracts and torts. e. extends to purchases and sale of real estate.

a Diff: 2 Type: MC Topic: Minors (or Infants

Minors will be liable for contracts a. if the contracts concern necessaries. b. if the minor completes a declaration stating that he or she has read and understood the terms of the contract. c. if they signed the contract. d. if the parent of the minor was present at the time the minor was signing the contract. e. if the minor is deemed to be of well-above-average intelligence.

a Diff: 2 Type: MC Topic: Minors (or Infants

Upon attaining the age of majority, a minor may be bound by two kinds of voidable contracts, namely, a. contracts that create an interest of a continuous and permanent nature, and contracts that are ratified by the minor. b. contracts pertaining to real estate, and contracts pertaining to intellectual property. c. contracts of service and contracts of employment d. contracts of purchase, and contracts of sale. e. contracts that concern necessaries, and contracts that concern non-necessaries.

a Diff: 2 Type: MC Topic: Minors (or Infants

Use the fact situation in Q8 to answer the related question that follows. In the fact situation above, Harry is a. an occupier. b. a licensee. c. a trespasser. d. an invitee. e. a contractual entrant.

a Diff: 2 Type: MC Topic: Occupier's Liability Skill: Applied 12

Jack decides to sue Mary and consults a lawyer. Because Jack had very little money, the lawyer agrees that his fees will take the form of a percentage of the damages Jake collects if he wins. The lawyer's fee arrangement is a a. contingent fee arrangement. b. flat fee arrangement. c. solicitor-client fee arrangement. d. sliding scale fee arrangement. e. party and party costs.

a Diff: 2 Type: MC Topic: Procedural Law: Using the Courts Skill: Recall 28

When we say that in order to institute a lawsuit, a person must first have standing, we mean that a. the person bringing the lawsuit must establish that he or she has the right to bring the lawsuit. b. the person bringing the lawsuit might be someone who has been wronged. c. the person defending the lawsuit must establish that he or she has a good defence to it. d. must be able to stand her or his ground during it. e. the person bringing the lawsuit must explain to the court what his or her lawsuit is about.

a Diff: 2 Type: MC Topic: Procedural Law: Using the Courts Skill: Recall/Applied 12

Mary works at a bread-making factory. All the women workers at the plant are paid lower wages than their male counterparts for the same work. The women workers decide to sue. They decide that Mary should represent all of them in a lawsuit. This kind of lawsuit is known as a. a class action. b. a labour dispute. c. res judicata action. d. a plaintiff claim. e. a representative action.

a Diff: 2 Type: MC Topic: Procedural Law: Using the Courts Skill: Recall/Applied 30

Which of the following would a court NOT regard as consideration on the part of a party to a contract? a. a promise to pay for what the other party has already done b. giving up what the party honestly believes is a right to sue c. a promise to pay money to the other party in the future d. a promise to pay money to a third party who is not a party to the contract e. money paid to the other party

a Diff: 2 Type: MC Topic: The Meaning of Consideration Skill: Applied 3

Consideration is a. the price that is paid for the promise of another. b. a special kind of gratuitous promise. c. the acceptance of an offer. d. the offer that is made to another. e. a special kind of acceptance.

a Diff: 2 Type: MC Topic: The Meaning of Consideration Skill: Recall 5

Use the fact situation in Q3 to answer the related question that follows: In the above fact situation, acceptance occurs a. when the cashier takes Alexander's money and hands the tea to Alexander. b. when Alexander takes the tea off the shelf. c. when Alexander takes the tea home. d. when Alexander hands the money to the cashier. e. when the cashier says goodbye to Alexander.

a Diff: 2 Type: MC Topic: The Nature of an Offer Skill: Recall/Applied 6

Law created by administrative agencies that are authorized by statute to make laws for certain purposes is called a. subordinate legislation. b. common law. c. law merchant. d. administrative law. e. domestic law.

a Diff: 2 Type: MC Topic: The Sources of Law Skill: Recall 22

The Hedley Byrne case establishes the principle of liability to ________ for ________. a. third parties; negligence misrepresentation b. clients; breach of fiduciary duty c. third parties; tort d. friends and relatives; negligence e. clients; breach of contract

a Diff: 2 Type: MC Topic: Tort Liability for Inaccurate Statements Skill: Recall 20

During parliamentary debate one day, James describes Peter as " a thief and a liar." Peter decides to sue for defamation. James has a defence of absolute privilege. a. True Correct: Correct b. False Incorrect: Incorrect

a Diff: 2 Type: TF Topic: Intentional Torts Skill: Applied 54

The literal approach to the interpretation of statutes requires a consideration of the plain meaning of the provision of a statute. a. True Correct: Correct b. False Incorrect: Incorrect

a Diff: 2 Type: TF Topic: Challenging the Validity of a Statute Skill: Recall 46

Where a contract is illegal, the contract exists, but will not be enforced by the court. a. True Correct: Correct b. False Incorrect: Incorrect

a Diff: 2 Type: TF Topic: Contracts Illegal by the Common Law and Public Policy Skill: Recall/Applied 34

There is no contract unless each party intends to create a legally enforceable contract. a. True Correct: Correct b. False Incorrect: Incorrect

a Diff: 2 Type: TF Topic: Intention to Create Legal Relations Skill: Recall 39

In some circumstances, there is immunity from defamation lawsuits. Words spoken in parliamentary debate, in proceedings in law courts and inquests, and before royal commissions, are subject to absolute privilege. a. True Correct: Correct b. False Incorrect: Incorrect

a Diff: 2 Type: TF Topic: Intentional Torts Skill: Applied 47

The roots of a tree that is growing on one property and that continue to grow onto another property may constitute a nuisance. a. True Correct: Correct b. False Incorrect: Incorrect

a Diff: 2 Type: TF Topic: Intentional Torts Skill: Applied 45

If a client sues a professional for failing to exercise reasonable care in giving advice, but the client has suffered no damage as a result, the professional will not be liable. a. True Correct: Correct b. False Incorrect: Incorrect

a Diff: 2 Type: TF Topic: Liability of Professionals Skill: Recall 40

Damages for breach of contract and breach of fiduciary duty are based on the same principle. a. True Correct: Correct b. False Incorrect: Incorrect

a Diff: 2 Type: TF Topic: Liability of Professionals Skill: Recall/Applied 41

An invitee is a person permitted by the occupier to enter the premises for business purposes, for example a shopper. a. True Correct: Correct b. False Incorrect: Incorrect

a Diff: 2 Type: TF Topic: Occupier's Liability Skill: Recall 50

Most Canadian jurisdictions have now abolished the distinction between invitees and licensees. a. True Correct: Correct b. False Incorrect: Incorrect

a Diff: 2 Type: TF Topic: Occupier's Liability Skill: Recall 51

The great majority of civil lawsuits do not proceed to trial because the parties ultimately settle. a. True Correct: Correct b. False Incorrect: Incorrect

a Diff: 2 Type: TF Topic: Procedural Law: Using the Courts Skill: Recall 48

Before a person can commence a suit, he or she must have standing to sue. a. True Correct: Correct b. False Incorrect: Incorrect

a Diff: 2 Type: TF Topic: Procedural Law: Using the Courts Skill: Recall/Applied 46

In some cases an otherwise blameless employer may be required at law to compensate victims of criminal acts committed by its employees. a. True Correct: Correct b. False Incorrect: Incorrect

a Diff: 2 Type: TF Topic: The Basis for Liability Skill: Recall 42

Once a contract has been made, legal capacity will be presumed unless the defendant proves otherwise. a. True Correct: Correct b. False Incorrect: Incorrect

a Diff: 2 Type: TF Topic: The Burden of Proving Essential Elements of a Contract Skill: Recall 30

When the offeror specifically states in the contract that acceptance can be given by performing an act, direct verbal communication of acceptance is not needed to make the contract binding. a. True Correct: Correct b. False Incorrect: Incorrect

a Diff: 2 Type: TF Topic: The Elements of Acceptance Skill: Recall 33

Unlike United States Supreme Court judges, Canadian Supreme Court judges are not subjected to a public and politicized confirmation process. a. True Correct: Correct b. False Incorrect: Incorrect

a Diff: 2 Type: TF Topic: The Role of Judges Skill: Recall 47

Administrative agencies derive their authority from regulations passed under statutes. a. True Correct: Correct b. False Incorrect: Incorrect

a Diff: 2 Type: TF Topic: The Sources of Law Skill: Recall/Applied 35

In any jurisdiction of Canada, such as Ontario, the rule is that the decision of a higher court is binding on a lower court. a. True Correct: Correct b. False Incorrect: Incorrect

a Diff: 2 Type: TF Topic: The System of Courts in Canada Skill: Recall 34

When an offer is proposed to an interested party, the form of acceptance does not matter even if a preferred method of communication is stipulated within the offer. a. True Correct: Correct b. False Incorrect: Incorrect

a Diff: 2 Type: TF Topic: Transactions Between Parties at a Distance from Each Other Skill: Recall/Applied 34

Jack and Jill agree to tell Michael that they will beat him up if he does not give them his valuable coin collection. According to the common law, this agreement between Jack and Jill is a. unenforceable. b. illegal. c. voidable. d. ineffective. e. void.

b Diff: 1 Type: MC Topic: Contracts Illegal by the Common Law and Public Policy Skill: Recall 17

The reasonable bystander test is a. a test that employs witnesses to make candid observations about contract cases. b. the test that is used by the courts to determine whether parties intended to enter into legal relations with each other. c. the test that employs non-specialists as members of a jury in contract cases. d. the test that the parties can use to determine who is in breach of contract. e. the test that the parties can use to determine compensation in contract cases.

b Diff: 1 Type: MC Topic: Intention to Create Legal Relations Skill: Recall 19

The primary goals of the common law when it was developed were a. efficiency and regularity. b. consistency and predictability. c. efficiency and speed. d. justice and speed. e. justice and regularity.

b Diff: 1 Type: MC Topic: Legal Systems: Civil Law and Common Law Skill: Recall 20

A disclaimer is a. a statement made by one person to the effect that the person making the statement will assume liability for loss up a certain specified limit. b. a statement to the effect that the person making it does not assume any responsibility for an action. c. an exclamation. d. a statement made in a soft voice or tone. e. a statement in a claim of a client against a professional to the effect that the client seeks to claim damages for specified breaches.

b Diff: 1 Type: MC Topic: Liability of Professionals Skill: Applied 25

Jack is 13 years old and a young musical legend. Jack enters into negotiations with a studio pertaining to a starring role in a series of movies. Prior to the negotiations, and as a condition to entering into the negotiations, Jack and the studios enter into an agreement under which the studio agrees to buy Jack a limousine for his personal use. The studio buys the limousine. A few months later Jack repudiates the contract and his chauffeur has an accident with the limousine. Which of the following is true? a. Jack may sell the limousine and then pay the proceeds to the studio. b. Jack will have to return the limousine in the state it is in. c. The studio can sue Jack for breach of contract. d. Jack may keep the limousine but has to pay for it. e. Jack will have to repair the limousine and then return it.

b Diff: 1 Type: MC Topic: Minors (or Infants

When Mary was 17 years old, she was asked to become a partner in an clothing design shop because she created brilliant clothing designs. Mary liked the idea at the time and signed a partnership agreement/contract. However, after working for a month, she decided that it was not something she wanted to do. After Mary turned 18, she decided to stop being a partner in the clothing shop. In this situation, a. Mary can repudiate the partnership contract because it is not a beneficial contract of service. b. so long as Mary does not affirm or ratify the partnership contract, she can repudiate it. c. there is nothing that Mary can do because she is now 18 years of age. d. there is nothing that Mary can do because she is bound by a beneficial contract of service. e. there is nothing Mary can do because she signed the partnership agreement/contract.

b Diff: 1 Type: MC Topic: Minors (or Infants

A offers to sell his car to B for $6000.00. However, B replies, "I will not buy it at that price," and walks away, never to be heard from again. In this situation, B's reply constitutes a. a lapse of A's offer. b. a rejection of A's offer. c. a revocation of A's offer. d. a counter-offer. e. none of the above

b Diff: 1 Type: MC Topic: Rejection and Counter-offer by the Offeree Skill: Recall 19

Exemplary damages are awarded by the court when it intends to a. grant an injunction. b. punish the wrongdoer. c. compensate the victim for out-of-pocket expenses that can be quantified. d. provide compensation for intangible injuries, such as pain and suffering. e. compensate the victim's family for the loss of their loved one.

b Diff: 1 Type: MC Topic: Remedies Skill: Recall 33

Silence can be a sufficient method of acceptance a. when the contract is a bilateral contract. b. if the parties to a contract have habitually used this method to communicate assent. c. when the offeror leaves the jurisdiction after making the offer. d. when the contract is a unilateral contract. e. when the offeree forgot to communicate its acceptance.

b Diff: 1 Type: MC Topic: The Elements of Acceptance Skill: Applied 30

An offer may be revoked before it is accepted. To be valid, a revocation must be a. promised. b. communicated to the offeree before acceptance. c. subject to a condition. d. loud and clear. e. exclusive.

b Diff: 1 Type: MC Topic: The Lapse and Revocation of an Offer Skill: Recall 13

Capacity to contract means a. the ability to make an offer or accept an offer. b. the competence to enter into a legally binding contract. c. the ability to pay damages in the event of a breach of contract. d. the ability to make a promise. e. the ability to comprehend the terms of a contract.

b Diff: 1 Type: MC Topic: The Meaning of Capacity to Contract Skill: Recall 2

Although there has been an offer and acceptance and consideration, and the parties intended to enter into a contract, a court will set aside a contract as void because of a. the social standing of one of the parties. b. the lack of capacity of one of the parties. c. the wealth of one of the parties. d. the signature on the contract being unclear. e. the impecuniosity of one of the parties.

b Diff: 1 Type: MC Topic: The Meaning of Capacity to Contract Skill: Recall 3

An offer is made by an ________ to an ________. a. inviter; invitee b. offeror; offeree c. initiator; offeree d. initiator; receiver e. offeror; invitee

b Diff: 1 Type: MC Topic: The Nature of an Offer Skill: Recall 8

A misrepresentation is a. a false assertion of fact made without knowledge of its falsity, and with honest belief in its truth. b. a false assertion of fact made with knowledge of its falsity, or without honest belief in its truth. c. a false assertion of fact about another person that defames that person. d. a statement of fact that was true when it was made, but later became false due to changes in circumstances. e. a missed representation.

b Diff: 1 Type: MC Topic: Tort Liability for Inaccurate Statements Skill: Recall 16

In examining contracts that include a restrictive covenant, the court will not presume that the term is in restraint of trade unless the evidence establishes that it is. a. True Incorrect: Incorrect b. False Correct: Correct

b Diff: 1 Type: TF Topic: Agreements in Restraint of Trade Skill: Recall 40

Promissory estoppel can be used as the basis for a lawsuit. a. True Incorrect: Incorrect b. False Correct: Correct

b Diff: 1 Type: TF Topic: Equitable Estoppel Skill: Recall 38

An attack by one person on the reputation of another person constitutes the tort of injurious falsehood. a. True Incorrect: Incorrect b. False Correct: Correct

b Diff: 1 Type: TF Topic: Intentional Torts Skill: Recall/Applied 46

The law is central to a business's interactions with its customers, suppliers, competitors, and government. a. True Incorrect: Incorrect b. False Correct: Correct

b Diff: 1 Type: TF Topic: Law and Business Skill: Recall/Applied 44

To run a successful business, the owners and managers of that business are not required to know the law. a. True Incorrect: Incorrect b. False Correct: Correct

b Diff: 1 Type: TF Topic: Legal Risk Management Skill: Recall 39

English common law is based on laws that are codified. a. True Incorrect: Incorrect b. False Correct: Correct

b Diff: 1 Type: TF Topic: Legal Systems: Civil Law and Common Law Skill: Recall 33

The duty of a professional to take reasonable care does not include a duty not to omit essential matters. a. True Incorrect: Incorrect b. False Correct: Correct

b Diff: 1 Type: TF Topic: Liability of Professionals Skill: Recall 39

An occupier has no liability whatsoever to a trespasser. a. True Incorrect: Incorrect b. False Correct: Correct

b Diff: 1 Type: TF Topic: Occupier's Liability Skill: Recall/Applied 38

A statement of defence cannot be combined with a counterclaim. a. True Incorrect: Incorrect b. False Correct: Correct

b Diff: 1 Type: TF Topic: Procedural Law: Using the Courts Skill: Recall 43

An acceptance must always be communicated to the offeror for a valid contract to arise. a. True Incorrect: Incorrect b. False Correct: Correct

b Diff: 1 Type: TF Topic: The Communication of an Offer Skill: Recall/Applied 39

Legal capacity simply refers to the ability of a party to enter into a contract. a. True Incorrect: Incorrect b. False Correct: Correct

b Diff: 1 Type: TF Topic: The Meaning of Capacity to Contract Skill: Recall 31

An invitation to do business is a commitment to enter into a contract upon acceptance. a. True Incorrect: Incorrect b. False Correct: Correct

b Diff: 1 Type: TF Topic: The Nature of an Offer Skill: Recall 37

Goods displayed on shelves in a store constitute an offer. a. True Incorrect: Incorrect b. False Correct: Correct

b Diff: 1 Type: TF Topic: The Nature of an Offer Skill: Recall 40

Equity is currently separate from the common law. a. True Incorrect: Incorrect b. False Correct: Correct

b Diff: 1 Type: TF Topic: The Sources of Law Skill: Recall 42

In Canada, only the federal and provincial governments make the laws. a. True Incorrect: Incorrect b. False Correct: Correct

b Diff: 1 Type: TF Topic: Who Makes Law? Skill: Recall 42

The two models of legal aid that are used in Canada are the a. Ontario legal aid and legal clinics. b. community legal clinic and judicare. c. judicare and payment plans. d. payment plans and community clinic. e. free services and community legal services.

b Diff: 2 Type: MC Topic: Procedural Law: Using the Courts Skill: Recall/Applied 29

Section 15 of the Canadian Charter of Rights and Freedoms deals with a. its application to citizens of the United States. b. equality rights. c. national politics. d. the applicability of the Human Rights Act. e. legal rights.

b Diff: 2 Type: MC Topic: The Charter of Rights and Freedoms Skill: Applied 27

The reasonable amount that a person deserves to be paid for goods and services provided to the person requesting them is called a. gratuitous promise. b. quantum meruit. c. damages. d. covenant. e. promissory amount.

b Diff: 2 Type: MC Topic: The Effect of a Request for Goods or Services Skill: Applied 23

Mary goes into a store in which she sees a SALE sign. While she is looking round, she notices that a sign that says 50 percent off has been placed against a pair of knee-high boots. She takes the pair of boots to the cashier. The cashier refuses to sell the boot at the 50 percent discount. Which of the following is true? a. The cashier is in breach of contract. b. Mary is not entitled to a 50 percent discount because the sign was not an offer. c. The conduct of the store amounts to false advertising and Mary will be entitled to the 50 percent discount. d. Mary is entitled to a 50 percent discount because that boots were advertized as being sold at a discount of 50 percent. e. The store is in breach of contract.

b Diff: 2 Type: MC Topic: The Nature of an Offer Skill: Recall/Applied 7

We study business contract law in order to a. know the consequences of breaking one's obligation. b. understand and bargain for contractual obligations. c. resolve commercial disputes. d. understand legislation that pertains to business. e. know when a binding contract is entered into.

b Diff: 2 Type: MC Topic: The Role of Contract Law Skill: Recall/Applied 31

A simple definition of law would be misleading because law is so a. bound up with the rule of law. b. diverse and complex. c. difficult to define. d. none of the above e. all of the above

b Diff: 2 Type: MC Topic: The Role of Law Skill: Recall 2

When it comes to interpreting statutes, a precedent is formed when courts a. decide on the meaning of the language of the statute. b. decide what the language of the statute means; whether it applies to the facts of a case; and, if it does, its consequences. c. determine whether the statute applies to the facts of a case. d. decide on the consequences of the statute when applied to a particular case. e. use and apply an existing case to the case being dealt with.

b Diff: 2 Type: MC Topic: The Sources of Law Skill: Recall 9

Which of the following is NOT/are NOT a Federal Court of Canada? a. Federal Court of Canada b. Federal Family Court c. Supreme Court of Canada d. Tax Court e. all of the above

b Diff: 2 Type: MC Topic: The System of Courts in Canada Skill: Recall 24

Which of the following statements best explains the standard of care in the law of tort? a. The standard is one that the judge expects. b. The standard is one that differs from person to person and activity to activity. c. The standard is one that society expects. d. The standard is one that the lawyer expects. e. The standard is the same for all persons and activities.

b Diff: 2 Type: MC Topic: Unintentional Torts Skill: Recall 27

Under our legal system, which is derived from English common law, a. our courts simply resolve the dispute based on common sense. b. our courts look to and apply principles of law that have evolved out of cases dealing with the same subject matter as does the dispute between the private parties. c. our courts look to the way other countries have resolved disputes between private parties. d. our courts look to legislation passed by the federal and provincial governments to resolve disputes between private parties. e. our courts look to the constitution to resolve disputes between private parties.

b Diff: 2 Type: MC Topic: Who Makes Law? Skill: Recall 17

Malicious prosecution concerns the prosecution of crimes and is not a tort. a. True Incorrect: Incorrect b. False Correct: Correct

b Diff: 2 Type: TF Topic: Intentional Torts Skill: Recall 53

Quantum meruit does not refer to the reasonable amount that a person merits for goods and services provided. a. True Incorrect: Incorrect b. False Correct: Correct

b Diff: 2 Type: TF Page Reference: 125 Topic: The Effect of a Request for Goods or Services Skill: Recall 40

Generally speaking, it is up to the court to ensure that a fair bargain was made; that is, to ensure that the consideration for a promise is adequate. a. True Incorrect: Incorrect b. False Correct: Correct

b Diff: 2 Type: TF Topic: Adequacy of Consideration Skill: Recall 33

Where a contract contains a restrictive covenant, that is, a covenant in restraint of trade, the totality of the contract is invalid. a. True Incorrect: Incorrect b. False Correct: Correct

b Diff: 2 Type: TF Topic: Agreements in Restraint of Trade Skill: Recall/Applied 37

ABC Inc. and XYZ Ltd. enter into an agreement. The agreement includes, among other things, a provision that both parties will monitor their prices to ensure that both parties price their products equally. In this situation, the court will set aside the whole contract as being illegal. a. True Incorrect: Incorrect b. False Correct: Correct

b Diff: 2 Type: TF Topic: Agreements in Restraint of Trade Skill: Recall/Applied 39

An arbitration is presided over by a mediator whose decision is binding on the parties to the arbitration. a. True Incorrect: Incorrect b. False Correct: Correct

b Diff: 2 Type: TF Topic: Alternative Dispute Resolution Skill: Recall 38

Within a standard form contract there always exists the ability to bargain the terms of the contract. a. True Incorrect: Incorrect b. False Correct: Correct

b Diff: 2 Type: TF Topic: An Offer Made by Tendering a Written Document to the Offeree Skill: Applied 32

The essence of causation in professional-client relationships is the suffering of harm or damage. a. True Incorrect: Incorrect b. False Correct: Correct

b Diff: 2 Type: TF Topic: Causation Skill: Recall/Applied 32

Equitable remedies such as specific performance originated in the early common law courts. a. True Incorrect: Incorrect b. False Correct: Correct

b Diff: 2 Type: TF Topic: Common Law: The Theory of Precedent Skill: Recall 41

A high intensity sound coming from one property and affecting other properties does not constitute the tort of nuisance. a. True Incorrect: Incorrect b. False Correct: Correct

b Diff: 2 Type: TF Topic: Intentional Torts Skill: Applied 44

Like a corporation, a social club has legal standing to commence and maintain an action at law. a. True Incorrect: Incorrect b. False Correct: Correct

b Diff: 2 Type: TF Topic: Labour Unions Skill: Recall 33

In commerce today, businesses have no need to live up to higher ethical standards. In other words, businesses need not consider the concept of corporate responsibility. a. True Incorrect: Incorrect b. False Correct: Correct

b Diff: 2 Type: TF Topic: Law and Business Ethics Skill: Applied 41

Developing a legal risk management plan is the best way to ensure that a business is managed and run properly. a. True Incorrect: Incorrect b. False Correct: Correct

b Diff: 2 Type: TF Topic: Legal Risk Management Skill: Recall 40

Where two professionals have specialized knowledge and skill in the same profession, representation of one by the other negates the usual standard of care for the professional who is representing the other. a. True Incorrect: Incorrect b. False Correct: Correct

b Diff: 2 Type: TF Topic: Liability of Professionals Skill: Applied 35

If the plaintiff has suffered no damages, a defendant who has breached a fiduciary duty will not be held liable. a. True Incorrect: Incorrect b. False Correct: Correct

b Diff: 2 Type: TF Topic: Liability of Professionals Skill: Recall 38

Motive can change a gratuitous promise into a binding contract. a. True Incorrect: Incorrect b. False Correct: Correct

b Diff: 2 Type: TF Topic: Motive Contrasted with Consideration: Past Consideration Skill: Recall 32

Most legal cases do not go to trial because the plaintiffs simply abandon their rights. a. True Incorrect: Incorrect b. False Correct: Correct

b Diff: 2 Type: TF Topic: Procedural Law: Using the Courts Skill: Recall 47

Compensation in tort law is not always in the form of damages. a. True Incorrect: Incorrect b. False Correct: Correct

b Diff: 2 Type: TF Topic: Remedies Skill: Recall/Applied 39

Restitution is a remedy available to a plaintiff who seeks an order restraining a person from doing, or continuing to do, a particular act. a. True Incorrect: Incorrect b. False Correct: Correct

b Diff: 2 Type: TF Topic: Remedies Skill: Recall 55

Under strict liability, a wrongdoer is punished strictly if the wrongdoer was at fault. a. True Incorrect: Incorrect b. False Correct: Correct

b Diff: 2 Type: TF Topic: The Basis for Liability Skill: Recall 48

An employer who has expressly prohibited certain actions can never be vicariously liable if an employee ignores this prohibition. a. True Incorrect: Incorrect b. False Correct: Correct

b Diff: 2 Type: TF Topic: The Basis for Liability Skill: Recall 43

If a statute states that it "shall operate notwithstanding" certain rights, this means that permission of parliament is needed to infringe on Charter rights. a. True Incorrect: Incorrect b. False Correct: Correct

b Diff: 2 Type: TF Topic: The Charter of Rights and Freedoms Skill: Recall 33

The rights prescribed in the Charter of Rights and Freedoms are absolute. a. True Incorrect: Incorrect b. False Correct: Correct

b Diff: 2 Type: TF Topic: The Charter of Rights and Freedoms Skill: Recall 45

An offer will lapse only if the offeree has had a reasonable time to accept. a. True Incorrect: Incorrect b. False Correct: Correct

b Diff: 2 Type: TF Topic: The Lapse and Revocation of an Offer Skill: Recall 41

The law reflects the highest level of moral and ethical standards that are universally accepted by all people. a. True Incorrect: Incorrect b. False Correct: Correct

b Diff: 2 Type: TF Topic: The Role of Law Skill: Applied 36

Normal standards of care always apply to professionals who undertake a task beyond the usual skills of the profession. a. True Incorrect: Incorrect b. False Correct: Correct

b Diff: 2 Type: TF Topic: The Standard of Care for Professionals Skill: Recall/Applied 31

A compliance officer employed by a business for purposes of monitoring legislative requirements applicable to the business ensures a. that outside counsel are hired by the business on a general retainer basis to respond to urgent matters. b. that the business complies with all regulatory and legislative requirements to avoid claims, fines and other possible penalties or sanctions. c. that probationary employees are screened and monitored appropriately by the business in the 90 day period. d. that the business pays outstanding fines. e. that someone attends on all Small Claims Court matters where outside counsel are not required.

b Diff: 3 Type: MC Topic: Legal Risk Management Skill: Applied 7

Where a professional undertakes a task that is beyond the usual skills of his or her profession, a. the professional must still follow the recommended standard. b. the professional must use the degree of skill and knowledge commensurate with the task undertaken. c. the professional must follow a well-known practice. d. the professional must follow all standards suggested as the proper standard. e. the professional need not follow any standard.

b Diff: 3 Type: MC Topic: Liability of Professionals Skill: Recall/Applied 12

Prior to the legislative changes in Canada with respect to contributory negligence, if a plaintiff contributed in any way to her or his own loss the action would fail. Statutes now require courts to a. determine if the plaintiff's degree of fault exceeds the defendant's degree of fault as a pre-condition to dismissing the action. b. apportion damages according to the respective degree of responsibility of the parties. c. find against the most liable defendant. d. follow strictly the responsibility chart set out in the statute.

b Diff: 3 Type: MC Topic: Unintentional Torts Skill: Recall 35

In a federal country such as Canada, the Supreme Court of Canada, and not Parliament, usually has the last word on the law because a. the judges of the Supreme Court of Canada are appointed by the federal government. b. there are two distinct levels of government. c. the Supreme Court of Canada is the highest court in the land. d. the Supreme Court of Canada is made up of judges who make law. e. Parliament only makes laws.

b Diff: 3 Type: MC Topic: Who Makes Law? Skill: Applied 21

Law is derived from a variety of sources. These include the constitution, legislation, and a. the Supreme Court of Canada and subordinate legislation. b. court decisions handed down by judges. c. statements made by ministers and administrative rulings. d. media reports and other news. e. the cabinet.

b Diff: 3 Type: MC Topic: Who Makes Law? Skill: Applied 23

Where there is an existing contract between two parties and, to ensure performance, one party promises to give more consideration than under the original contract, the new promise is binding and must be performed. a. True Incorrect: Incorrect b. False Correct: Correct

b Diff: 3 Type: TF Topic: Adequacy of Consideration Skill: Applied 34

A court will refuse to enforce a contract that involves illegality only if it can be shown that both parties knowingly agreed to illegality. a. True Incorrect: Incorrect b. False Correct: Correct

b Diff: 3 Type: TF Topic: Contracts Illegal by the Common Law and Public Policy Skill: Applied 36

An inmate is able to escape from a local prison because a security guard left a gate open. The jail owes the same duty of care to a local farmer who is assaulted by the inmate minutes later as it does to a gas station owner who operates 750 kilometres away and is robbed three months later. a. True Incorrect: Incorrect b. False Correct: Correct

b Diff: 3 Type: TF Topic: Duty - Unintentional Torts Skill: Applied 52

All gratuitous promises are unenforceable. a. True Incorrect: Incorrect b. False Correct: Correct

b Diff: 3 Type: TF Topic: Gratuitous Promises Skill: Recall/Applied 31

A contract made by a minor is enforceable against him or her, and unenforceable by him or her. a. True Incorrect: Incorrect b. False Correct: Correct

b Diff: 3 Type: TF Topic: Minors (or Infants

Like acceptance, revocation of an offer by mail is valid and binding once the letter of revocation is placed in the mailbox. a. True Incorrect: Incorrect b. False Correct: Correct

b Diff: 3 Type: TF Topic: The Elements of Acceptance Skill: Recall/Applied 36

The privilege between a doctor and his or her patient is recognized by law in the same way as solicitor-client privilege. a. True Incorrect: Incorrect b. False Correct: Correct

b Diff: 3 Type: TF Topic: The Legal Profession Skill: Recall/Applied 37

A contract that is void is also unenforceable. a. True Incorrect: Incorrect b. False Correct: Correct

b Diff: 3 Type: TF Topic: The Meaning of Capacity to Contract Skill: Applied 32

Law and justice must, of necessity, coincide regardless of whether the result is morally right or Correct. a. True Incorrect: Incorrect b. False Correct: Correct

b Diff: 3 Type: TF Topic: The Role of Law Skill: Applied 37

The merger of the courts of chancery with the courts of common law has resulted in judges abandoning the philosophy of equity when deciding cases. a. True Incorrect: Incorrect b. False Correct: Correct

b Diff: 3 Type: TF Topic: The Sources of Law Skill: Recall/Applied 36

What is subordinate legislation?

Subordinate legislation is the rules that are passed by a body designated in a statute, pursuant to the provisions of that statute. Diff: 2 Type: ES Topic: Who Makes Law? Skill: Recall 48

Generally speaking, there are two reasons why a court will declare a statute invalid. State these two reasons.

1

What is meant by owing a "duty of care" in the tort of negligence?

A duty of care is said to be owed by one person, for example, Jack, to another, for example, Jim, where the closeness of Jack and Jim is such that Jack ought reasonably to have Jim in contemplation as being affected by his acts or omissions. Diff: 2 Type: ES Topic: Unintentional Torts Skill: Applied 66

Explain the notion of concurrent powers as it pertains to the federal system of government in Canada.

A federal system of government is different from a unitary system of government. In a unitary system of government, such as the system of government that exists in England, there is only one level of government that makes laws. In a federal system of government such as the systems of government of the United States and Canada, there are two levels of government, the federal government and the provincial or state governments, and law-making power is divided between these two levels of government. Each level of government has jurisdiction to make laws with respect to certain areas. Sometimes the power of the federal and provincial governments may overlap in certain areas. When this happens, it is said that the federal and provincial governments have concurrent jurisdiction. Diff: 3 Type: ES Topic: Who Makes Law? Skill: Applied 56

What is a fiduciary duty?

A fiduciary duty is a duty that is imposed on a person by virtue of a special relationship between the fiduciary and some other person. Diff: 1 Type: ES Topic: Liability of Professionals Skill: Recall 51

What is a fiduciary duty?

A fiduciary duty is a duty that is imposed on a person who stands in a special relationship of trust to another. Diff: 1 Type: ES Topic: Liability of Professionals Skill: Recall 47

Distinguish between mediation and arbitration.

A mediation is presided over by a mediator, whose job it is to try to resolve the dispute between parties by facilitating a settlement between them. Usually, the mediator is an expert in the area of law that applies to the dispute and he or she will use a number of established techniques to help the parties resolve their dispute. However, a mediator cannot make a decision that is binding on the parties. An arbitration is a form of proceeding that is presided over by an official called an arbitrator, who is also usually an expert in the area of law that applies to the dispute. But in an arbitration the parties have agreed in advance to be bound by the arbitrator's decision. During the arbitration, the arbitrator hears evidence from each of the parties and then renders a decision that is final and binding on the parties. None of the parties has a right of appeal unless it can be shown that the arbitrator made an error by exceeding or going beyond his or her authority. Diff: 2 Type: ES Topic: Alternative Dispute Resolution Skill: Recall/Applied 59

A minor's freedom from liability is limited to contract. Explain.

A minor is a person who has not reached the age of majority specified by provincial statute. The general rule is that a contract made by a minor is unenforceable against the minor but enforceable by the minor against the other party. This is without regard to whether the other person is aware that he or she is dealing with a minor. However, the freedom of a minor from liability is limited to contract only. A minor remains liable for torts, such as negligence, assault, defamation, or deceit. Thus if a minor is negligent, he or she will be liable. However, if a minor is doing certain acts that are contemplated by a contract and loss results, the court will decide the matter under contract law and the minor will escape liability for the loss. Diff: 2 Type: ES Topic: Minors (or Infants

For the purpose of a contract, who is a minor?

A minor is someone who has not reached the age of majority specified in provincial legislation. Diff: 1 Type: ES Topic: Minors (or Infants

What is a multi-disciplinary partnership?

A multi-disciplinary partnership is one in which partners of one profession practice together with members of other professions, for example, lawyers practicing together with chartered accountants. Diff: 1 Type: ES Topic: Multi-disciplinary Partnerships Skill: Recall 46

What is a negative option?

A negative option is a provision in a contract that provides that if a party does not provide notice before its current term runs out, the contract is automatically renewed for another term. Diff: 1 Type: ES Topic: The Lapse and Revocation of an Offer Skill: Recall 45

What is the difference between negligent misrepresentation and fraudulent misrepresentation?

A negligent misrepresentation is an incorrect statement that is made without due care for its accuracy. A fraudulent misrepresentation is an incorrect statement made knowingly with the intention of causing injury to the other person. Diff: 2 Type: ES Topic: Tort Liability for Inaccurate Statements Skill: Recall/Applied 50

Why is a promise to perform a pre-existing legal duty not consideration?

A person who owes a pre-existing legal duty cannot rely on that duty as consideration for a new contract. The reasoning for this is sometimes said to be that a person cannot be made to pay twice for the same thing. It is also said to be against public policy for a person to be able to threaten to break one contract unless he or she is given another contract at a higher price. In essence however, the promise to perform a pre-existing legal duty is not consideration because the consideration for the pre-existing legal duty has already been provided. Diff: 2 Type: ES Topic: Relation Between Existing Legal Duty and Consideration Skill: Recall/Applied 52

Where there is negligence, the injured party may be required to mitigate his or her damages. Explain.

A person who suffers an injury due to the negligence of someone else has a duty to minimize his or her injuries where this is possible. Minimizing these injuries is a form of reducing or mitigating the injury and, thus, the damages caused by the negligence. Diff: 2 Type: ES Topic: Unintentional Torts Skill: Applied 60

There are cases in which a remedy other than damages may be available. What is the difference between a restitutionary order and an injunction?

A plaintiff attempting to recover property wrongfully converted by a defendant can obtain a restitutionary order requiring its return. Courts may also grant injunctive relief preventing a defendant from committing wrongful acts under threat of contempt of court. Diff: 3 Type: ES Topic: Remedies Skill: Applied 65

A professional must not place him or herself in a position where his or her interest and duty conflict. Explain.

A professional person is usually said to have a fiduciary obligation towards his or her clients. He or she is a fiduciary. A fiduciary is a person who has a special relationship of trust with another. The fiduciary is usually someone in a more powerful position vis-à-vis the other party. The nature of this power relationship imposes an obligation of utmost trust on the part of the fiduciary. That is, the fiduciary must be completely loyal to the other party. One of the consequences of this obligation is the duty of the fiduciary or professional not to place himself in a position that will give rise to a conflict of interest. That is a position where his or her interest will conflict with his or her duty. Thus, a professional will usually not be expected to act on behalf of two persons who have competing interests because the professional will find it hard to fulfill his or her duty to both parties. Diff: 3 Type: ES Topic: Liability of Professionals Skill: Applied 52

A business can be managed and run in either a prudent and responsible manner or a stupid manner. Explain how a prudent and responsible businessperson would manage and run a new business that he or she has just began as compared to a person who was not prudent or responsible.

A prudent and responsible businessperson would first familiarize him- or herself with the law. Then he or she would develop a risk management plan that deals with the risks, prioritize them in relation to the business, and implement the plan. Further, he or she would regularly review and update the plan to take into consideration any change of laws or any new laws. Finally, in order to better manage the business, he or she would voluntarily implement a code of conduct by creating one or looking to see whether the business was already governed by one, as in the case of many professions. A businessperson who was neither prudent nor responsible would not do any of these things but would simply run the business as he or she chose, maximizing the legal risks affecting the business and exposing it to all sorts of liability. Diff: 3 Type: ES Topic: Legal Risk Management Skill: Applied 57

What is the effect of sealing a contractual document?

A seal is a mark that is placed on a written contract to indicate the party's intention to be bound by the terms of the document. When a document is sealed, a party indicates his or her intention to be bound by the terms of the contract even though the other party may not have given consideration. Diff: 1 Type: ES Topic: The Use of a Seal Skill: Recall 42

How is an appeal different from a trial?

A trial is the culmination of an action. The parties to the dispute bring all their evidence and version of the facts before the court. The difference between the version of the facts of both parties is usually very wide. The court's task is to sift through all the evidence presented to determine which evidence is admissible, which facts are credible, and which witnesses are credible. Ultimately, when the judge makes a finding, it is a finding that a person's story is more likely than that of the other party. An appeal is very different from a trial. An appeal is essentially a review of the trial. The appeal judges review the trial and evidence to determine whether certain errors were made. Errors could include errors of law, drawing wrong conclusions from the evidence, and misconstruing the evidence. The appeal judges do not re-try the whole matter. Diff: 1 Type: ES Topic: The System of Courts Skill: Recall/Applied 53

Trial judges hear evidence and appeal judges review evidence. Explain.

A trial is the culmination of an action. The parties to the dispute bring all their evidence and version of the facts before the court. The difference between the version of the facts of both parties is usually very wide. The court's task is to sift through all the evidence presented to determine which evidence is admissible, which facts are credible, and which witnesses are credible. Ultimately, when the judge makes a finding, it is a finding that a person's story is more likely than that of the other party. An appeal is very different from a trial. An appeal is essentially a review of the trial. The appeal judges review the trial and evidence to determine whether certain errors were made. Errors could include errors of law, drawing wrong conclusions from the evidence, and misconstruing the evidence. The appeal judges do not re-try the whole matter. Diff: 1 Type: ES Topic: The System of Courts in Canada Skill: Recall/Applied 61

What is a unilateral contract?

A unilateral contract is one in which one party makes an offer to the whole world, and the contract is accepted by a person stepping forward to perform an act or acts required by the terms of the offer. Diff: 1 Type: ES Topic: Unilateral and Bilateral Contracts Skill: Applied 47

Explain the difference between a contract that is void and one that is illegal.

A void contract is one that never comes into existence, while an illegal contract is one that exists but by reason of illegality will not be enforced by the court. Diff: 3 Type: ES Topic: The Difference Between a Void and an Illegal Contract Skill: Applied 43

Why should you not make an agreement to sell your car to a purchaser "for a fair price"?

Although the courts have a policy of making contracts effective whenever possible, they will not make a contract for the parties. If an offer is vague, the court may find that there was no offer at all, and neither party will be bound. Price is an essential element, and if there has been no agreement between the parties as to price, there will be no contract to enforce. The expression "fair price" is too vague. Diff: 2 Type: ES Topic: Uncertainty in the Wording of an Offer Skill: Applied 54

Explain how a trial judge might avoid the rule of stare decisis, or precedent, to ensure that justice was done in a particular case that came before her or him so as to change the law.

Although the trial judge in such a case would understand that she or he was technically bound to follow earlier decisions dealing with the same subject as the case before her or him and also that normally decisions of higher courts would be binding on her or him, it would be open to her or him to distinguish the facts of the case before her or him from the facts of earlier decisions by dwelling on minor differences between them. This would permit the trial judge to depart from the normal rules and adjust the law to the case before her or him by applying it differently than it was applied in previous or higher decisions. However, the real test of the correctness of the trial judge's decision would come on appeal to either a court of appeal or the Supreme Court of Canada. If these courts, more particularly the Supreme Court of Canada, upheld the trial judge's decision, then her or his departure from the normal rules would be confirmed and she or he would have effectively changed the common law. Diff: 3 Type: ES Topic: Common Law: The Theory of Precedent Skill: Applied 57

Explain whether judges are required to follow the principle of stare decisis.

Although traditionally judges strictly followed this rule, today they are only bound by decisions of higher courts, and even then, since no two sets of facts are identical in every respect, judges can and do distinguish the facts of the case in issue before them with the facts of earlier similar cases in order to avoid this rule. Diff: 3 Type: ES Topic: Common Law: The Theory of Precedent Skill: Applied 49

When can an adult recover money loaned to a minor?

An adult can recover money loaned to a minor if the money was used to purchase necessaries. Diff: 2 Type: ES Topic: Minors (or Infants

Not all offers must be communicated to the offeror for a valid contract to arise. Explain.

As a general rule, when a person makes an offer, the acceptance of the offer must be communicated to the offeror before a contract can arise. This is the general rule where a bilateral contract is concerned. That is a contract where both parties make promises to each other. In a unilateral contract, however, where one party makes a promise to whomever steps forward to accept the contract, the acceptance of the contract does not have to be communicated before a valid contract arises. For instance, if Jack says, "I'll pay $1000 to anyone who finds and returns my cat," all that is required is for Jim to find and return the cat and Jack will be obliged to pay the $1000. Jim does not have to notify Jack that he, Jim, intends to look for the cat. Diff: 2 Type: ES Topic: Unilateral and Bilateral Contracts Skill: Applied 50

The concept of foreseeability in tort law is considered by a court in determining the existence of a duty of care and as a threshold for the recovery of any damage suffered by an injured plaintiff. Explain the significance of "reasonably foreseeable" as it relates to both duty and damages, as well as the policy reasons why courts sometimes fail to recognize a duty, or fail to recognize that the plaintiff has met the threshold for recovery of damages.

As far as duty of care goes, the courts will not impose liability unless it was "reasonably foreseeable" that a negligent act or omission would cause potential injury or loss to the plaintiff. Judges often refer to this aspect of the establishment of a duty of care as "proximity." With respect to the recovery of damages, the court employs a "reasonable foreseeability test" to establish a threshold that a plaintiff must meet or exceed to recover damages. The court undertakes an objective assessment of what a reasonable defendant would foresee as likely injury to the average plaintiff. The plaintiff must satisfy the court that a person of "ordinary fortitude" would likely have suffered injury, failing which, causation at law is not established. Policy concerns considered by the court in either recognizing a duty of care or recognizing an entitlement to claim damages include the proliferation of lawsuits and the ability to insure against a potential new risk. Diff: 3 Type: ES Topic: Unintentional Torts Skill: Applied 77

Nan promises not to sue Reg if he agrees to pay her $1000. Reg agrees to pay Nan $1000, but later discovers that she actually had no right to sue him. Is Reg bound to pay Nan?

As long as a party has an honest belief in a right to sue, giving up that right can be consideration for a promise on the part of the other party. This applies even though the belief in the right to sue was mistaken. Diff: 2 Type: ES Page Reference: 116 Topic: The Meaning of Consideration Skill: Recall 47

What is meant by the term contractual capacity?

Contractual capacity refers to the competence to enter into a contract. Diff: 1 Type: ES Topic: The Meaning of Capacity to Contract Skill: Recall 41

What is contributory negligence?

Contributory negligence is the negligence of an injured party that contributes to the injured party's own injury. Diff: 2 Type: ES Topic: Unintentional Torts Skill: Recall 68

Foreseeability is a major element in the determination of the extent of liability. Explain.

Foreseeability is the legal term that means the extent to which an injury could have been anticipated. That is to say, was the injury foreseeable. The law places a duty on all persons to not engage in acts, and to avoid omissions, that will result in foreseeable injury to another. Until the 1960s, the general principle was that a wrongdoer was liable for any injury that resulted directly from his or her actions, however unlikely it was that the injury could have flowed from the act or omission. In one of the landmark cases in the common law, The Wagon Mound, decided in the United Kingdom, the Court restricted liability to injury or damage that was reasonably foreseeable. In Canada, the current position takes a broad view of what is foreseeable. The position is that a wrongdoer takes a victim as he or she finds him or her. Thus if a victim has a pre-existing condition or disability such that the injury that flows from the act of the wrongdoer is far more serious than might have been expected, the wrongdoer will nevertheless be liable for the injury. Diff: 2 Type: ES Topic: Unintentional Torts Skill: Applied 76

Which is easier to prove, fraudulent or negligent misrepresentation? Why?

Fraudulent misrepresentation requires proving at least some guilty knowledge or willful disregard for the falsity of the information provided, and this may therefore be somewhat difficult to prove, while negligent misrepresentation requires only a breach of duty of care and skill, making it easier to prove. Diff: 3 Type: ES Topic: Tort Liability for Inaccurate Statements Skill: Applied 44

Distinguish between general and special damages.

General damages are intended to compensate an injured party for intangible injuries, such as pain and suffering. Special damages are intended to reimburse the injured for out-of-pocket expenses, such as hospital bills. Diff: 1 Type: ES Topic: Remedies Skill: Recall 63

Distinguish between general and punitive damages.

General damages are intended to compensate the injured party for intangible injuries, such as pain and suffering. Punitive damages are imposed by the court to punish the wrongdoer. Diff: 1 Type: ES Topic: Remedies Skill: Recall 64

Jack owns a pub. One evening, Jim, a friend, comes into the pub and orders a drink. Over a period of approximately two hours, Jim orders and drinks a variety of alcoholic beverages. Shortly before the pub closes, Jim pays for his drinks and leaves the pub. Jim gets into his car and drives away. On his way home, Jim is involved in a motor vehicle accident. Explain the nature of the tort that Jack would be found to have committed. Are there any defences available to Jack? Explain what these defences are.

Jack would most likely be found to have been negligent. The tort of negligence is committed when a party who is said to owe a duty of care to another breaches that duty. Jack may be said to owe a duty because he meets the test. Jack and Jim are in such proximity that Jack ought reasonably to consider Jim as being affected by his acts or omissions. In this instance, Jack owed a duty to Jim to stop Jim from drinking immediately once he determined that Jim was drunk. He also owed a duty to prevent Jim from driving home. Jack did not stop Jim, either from continuing to drink or from driving home, and this is in breach of his duty. As a result of Jack's omissions, Jim sustained an injury, which could be said to be directly related to Jack's omission. Jack is able to advance the defence of contributory negligence, and of voluntary assumption of risk, to reduce the amount of compensation to which Jim may be entitled. Jim was not only contributorily negligent, he also voluntarily assumed the risk that if he consumed too much alcohol and he drove home, he could be involved in an accident. Diff: 3 Type: ES Topic: Unintentional Torts Skill: Applied 74

Explain why a newspaper advertisement by a merchandising company is not a binding and enforceable contract.

Newspaper advertisements are invitations to do business. They are a device used to attract new customers and to start negotiations toward a contract. A customer may make an offer which the merchant can either refuse or accept, or the merchant may make an offer as soon as the customer shows interest. Newspaper advertisements are also invitations to the public to visit the merchant's place of business with a view to buying. This is because a business is not expected to sell the goods to everyone who reads its advertisements, only those who, having read an advertisement, either contact the merchant so as to begin negotiations towards a contract or actually go to the merchant for the same purpose. Diff: 2 Type: ES Topic: The Nature of an Offer Skill: Recall/Applied 43

Social policy influences judicial decision-making and legislative intervention in the area of tort law. Explain.

Society has certain standards by which it expects persons to live. Societal standards necessarily give rise to expectations regarding the consequences of a breach of those standards. In the area of tort law, societal standards give rise to expectations of how persons should act towards each other, and, in the event that one person injures another, the amount of compensation that should be paid to the injured party should the other party be deemed to be at fault. The expectations of society ultimately influence policy-making in the form of legislative intervention of judicial decision-making. Judges look at societal standards and expectations, for example, to determine how much a person who has lost a limb should be given in compensation. As well, lawmakers in the legislature make laws that reflect standards. For instance, in Canada, workplace safety laws require that employers contribute to workplace safety and insurance. An injured worker is paid from this pool of contributions. This law reflects the social policy that employers who do business should contribute to the rehabilitation of injured workers. Diff: 3 Type: ES Topic: The Basis for Liability Skill: Applied 73

Explain the relationship between the courts of common law and equity.

The common law system developed in feudal England at the time of the Norman conquest. The common law is said to be judge-made law because it is based on the recorded reasons of judges. At the core of the common law system is the theory of precedent, which means that judges should stand by the decided cases. The previous decisions of judges are thus important. Because of the importance of the theory of precedent and following previous decisions, the common law grew to be strict and inflexible and in most cases unjust. For instance, before a party could be heard, he or she had to ensure that his or her claim fell within a particular procedure, otherwise it would not be heard. Soon, it became necessary to modify the rigours of the common law. The king thus established another set of courts, referred to as courts of equity, which were intended to remedy the unfairness of the common law decisions. The courts of equity developed side by side with the common law courts and soon were administered by the Lord Chancellor. The Lord Chancellor was said to be the custodian of the conscience of the King. In 1865, the British Parliament passed an act that merged the two courts. From this time onwards, the same courts that administered the common law also administered the principles of equity. Hence the saying that equity and the common law go hand-in-hand. In Canada, various provinces have also passed acts that have merged the two systems of courts into one court. Diff: 2 Type: ES Topic: Legal Systems: Civil Law and Common Law Skill: Recall 60

Explain how the defence of qualified privilege may be used in a defamation action.

The defence of qualified privilege to a defamation action is one that is available to persons in positions of authority whose responsibility it is to provide information about other persons. Provided such persons give the information sought in good faith and with an honest belief in its accuracy, they can successfully defend against a defamation action even if the information proves to be false. Diff: 3 Type: ES Topic: Intentional Torts Skill: Recall 70

Should promissory or equitable estoppel only be used as a defence?

The doctrine of promissory or equitable estoppel originated as a bar to an action by one party who made a gratuitous promise that was relied on by and to the detriment of the party to whom the promise was made. Considering that this is much like the principle of negligent misrepresentation, which is actionable by a party, and also considering the decision of the Supreme Court of Canada in Conwest Exploration v. Letain, there is good reason to suggest that this doctrine can be used as the basis for a cause of action in a lawsuit. Diff: 3 Type: ES Topic: Equitable Estoppel Skill: Applied 49

A counter-offer by its very nature contains a rejection of the offer. Explain.

When a counter-offer is made, it is the same as saying no to the original offer and making a new offer in its place. In other words, by its very nature a counter-offer contains both a rejection and new offer. Diff: 2 Type: ES Topic: The Nature of an Offer Skill: Recall 42

Explain the position that a court will take when it is faced with a term in a contract that may be in restraint of trade.

When a court is faced with a term in a contract that is in restraint of trade, the court will presume that the term is in restraint of trade. The onus is then on the party seeking to enforce the covenant to rebut the presumption that the term is in restraint of trade. Diff: 1 Type: ES Topic: Agreements in Restraint of Trade Skill: Recall 45

What are the legal consequences of the ratification of a contract by a minor who attains the age of majority?

When a minor who attains the age of majority ratifies a contract, the minor acknowledges the contract and promises to perform his or her obligation under the contract. Diff: 2 Type: ES Topic: Minors (or Infants

What is an examination for discovery?

When an action is commenced, each party to the dispute has a right to question the other party, before trial, to determine the case that he or she has to answer and to determine the nature of the evidence that the other side possesses. These pre-trial question-and-answer sessions are known as examinations for discovery. Not all provinces provide for examinations for discovery in their court procedures. Diff: 1 Type: ES Topic: Procedural Law: Using the Courts Skill: Recall 55

What is meant by "the meeting of minds"? How significant is it to the formation of contracts?

When parties are entering into a contract, it is said that there must be a meeting of minds. That is to say, both parties must desire to enter into legal relations with each other on the same subject. Without the desire of both parties to a contract to enter into legal relations, a valid contract cannot be formed regardless of the fact that the other elements of the contract may be present. Diff: 2 Type: ES Topic: Relation Between Existing Legal Duty and Consideration Skill: Applied 1

Explain how the rule concerning withdrawal of an offer by post differs from that concerning the time of acceptance by post.

When the post is used to accept an offer, the general rule is that acceptance is complete when a properly addressed and stamped letter of acceptance is dropped in the mail. Regarding the withdrawal of an offer by post, the general rule is that the offer is revoked by post when the offeree actually receives notification of the revocation and not when it is dropped in the mail. Diff: 1 Type: ES Topic: Transactions Between Parties at a Distance from Each Other Skill: Applied 46

The fact that a labour union does not have capacity to sue and defend on its own is not an absolute bar to the bringing or defence of an action on its behalf. Explain.

While, subject to statute, a labour union may not itself have the capacity to bring a lawsuit, one of its members can bring a representative action on behalf of all members of the labour union because they all have the same interests. Diff: 3 Type: ES Topic: Labour Unions Skill: Recall/Applied 42

John is a member of a social club that does a great deal of charity work. Harry was once a member of the social club, but after an argument at one of the club meetings, Harry had his membership taken away. This made Harry very angry, and to get back at the club, Harry began to distribute flyers that told lies about the club's charity work and which caused people not to want to make any donations to the charities that the club sponsored. The club now wants to sue Harry for defamation. In this situation, a. the club is not a legal person in law and cannot sue Harry. b. the club is just like a labour union and will be able to sue Harry. c. the club is not a legal person in law, but it will still be able to sue Harry. d. the club is just like a corporation and will be able to sue Harry. e. none of the above

a Diff: 3 Type: MC Topic: Other Persons of Diminished Contractual Capacity Skill: Applied 26

Section 33 of the Charter allows the legislature to pass legislation that overrides the fundamental freedoms specified in the Charter only if a. the legislation expressly declares that it "shall operate notwithstanding" a provision included in the Charter. b. a court determines that the freedoms can be overridden. c. two-thirds of the provinces having at least 50 percent of the total population of Canada approve. d. the legislation in question does not violate the Human Rights Act. e. five years have passed since the legislation was first proposed.

a Diff: 3 Type: MC Topic: The Charter of Rights and Freedoms Skill: Applied 28

When a court interprets a statute narrowly, the result will most likely be that a. the statute will be found to fall inside the power of the government that made it. b. the court will change the wording in the statute so that it makes sense. c. the statute will be found to be outside, or ultra vires, the government that made it. d. the statute will be struck down. e. the court will send the statute back to the government that made it for revision.

a Diff: 3 Type: MC Topic: The Courts and Legislation Skill: Applied 15

Use the fact situation in Q8 to answer the related question that follows. In a lawsuit by Marty and Jonny against Harry for negligence, a. the fact that Marty was wearing shoes with leather soles may be found to be contributory negligence. b. the fact that Harry did not shovel his walk or put salt on it will likely not constitute negligence. c. the fact that Marty and Jonny should have known that Harry's walk was covered in ice is contributory negligence. d. the fact that Jonny was wearing rubber boots will likely be contributory negligence. e. the fact that Marty and Jonny should have known that Harry's walk was covered in ice is a voluntary assumption of risk.

a Diff: 3 Type: MC Topic: Unintentional Torts Skill: Applied 13

Phil was riding his motorcycle home from work when he was involved in a motor vehicle accident. As he lay on the road injured, he was attended to by two different motorists who stopped after seeing the accident. Additionally, an overhead helicopter from the local TV news station was broadcasting live on air. Suddenly, a second vehicle careened out of control and ran over Phil killing him instantly. Given concerns about proximity, foreseeability, and policy reasons, of all the people who suffered nervous shock, who would be most likely to recover compensation in a court action against the reckless driver of the second vehicle that caused Phil's death? a. Claims by the two motorists who were assisting Phil at the scene. b. Claims by the people who witnessed the incident on TV. c. Claims by the ambulance attendants who arrived 10 minutes after Phil died. d. Claims by those who read the story and saw pictures in the paper the following day.

a Diff: 3 Type: MC Topic: Unintentional Torts Skill: Applied 31

Where there is an existing contract between two parties and one party, to ensure performance by the other, promises something more, so long as the other party also gives something more, there is a binding contract and the first party must perform the new promise. a. True Correct: Correct b. False Incorrect: Incorrect

a Diff: 3 Type: TF Topic: Adequacy of Consideration Skill: Applied 36

After negotiation, alternative dispute resolution is the cheapest form of resolving disputes. a. True Correct: Correct b. False Incorrect: Incorrect

a Diff: 3 Type: TF Topic: Alternative Dispute Resolution Skill: Applied 39

The only court in Canada that is not bound by its own decisions is the Supreme Court of Canada. a. True Correct: Correct b. False Incorrect: Incorrect

a Diff: 3 Type: TF Topic: Common Law: The Theory of Precedent Skill: Applied 40

Estoppel or promissory estoppel was developed to overcome the principle that payment of a lesser sum in satisfaction of a greater sum is no payment at all. a. True Correct: Correct b. False Incorrect: Incorrect

a Diff: 3 Type: TF Topic: Equitable Estoppel Skill: Applied 35

An offer must be communicated to the offeree. a. True Correct: Correct b. False Incorrect: Incorrect

a Diff: 3 Type: TF Topic: Rejection and Counter-offer by the Offeree Skill: Applied 38

In any legal system, justice is merely the end result of that legal system and should not be equated with moral right and wrong. a. True Correct: Correct b. False Incorrect: Incorrect

a Diff: 3 Type: TF Topic: The Role of Law Skill: Applied 43

A simple definition of law would be misleading because law is so diverse and complex. a. True Correct: Correct b. False Incorrect: Incorrect

a Diff: 3 Type: TF Topic: The Role of Law Skill: Recall 34

When an offeror invites acceptance by mail, it is possible for the offeror to state in the offer that the offer will only become a contract once acceptance is actually received by the offeror. That is, the postal rule does not apply even though an invitation to acceptance by mail was given. a. True Correct: Correct b. False Incorrect: Incorrect

a Diff: 3 Type: TF Topic: Transactions Between Parties at a Distance from Each Other Skill: Recall/Applied 35

After a manufacturer places a product on the market, it has a continuing obligation to warn consumers of potential dangers. a. True Correct: Correct b. False Incorrect: Incorrect

a Diff: 3 Type: TF Topic: Unintentional Torts Skill: Recall 49

Use the fact situation in Q25 to answer the related question that follows. Review the fact situation above and now assume that on November 1, B said to A, "Rather than paying me the full $5000.00, just pay me $3500.00 and give me your watch, and your debt will be cancelled." A's watch is a cheap one worth $5.00. A gives B the watch and pays him the $3500.00. In this situation, a. B can recover the $1500.00 from A because the contract was to repay a debt of $3500.00. b. B cannot recover the $1500.00 from A because A's watch is fresh consideration. c. B is still estopped by promissory estoppel from collecting the $1500.00 from A. d. there is still no fresh consideration for B's promise to reduce A's debt to $3500.00. e. the watch is meaningless and A will still owe B $1500.00.

b Diff: 2 Type: MC Topic: Adequacy of Consideration Skill: Recall/Applied 28

At common law, a clause in an agreement between a purchaser and vendor for the sale and purchase of a business that provides that the vendor will not carry on the same business within a radius of 1500 kilometres of the business being sold a. is void for public policy. b. is not reasonable and amounts to a restraint of trade. c. is perfectly reasonable and will be enforced by the court. d. is an illegal clause and therefore void. e. none of the above

b Diff: 2 Type: MC Topic: Agreements in Restraint of Trade Skill: Applied 24

is in the business of shipping goods. Yesterday HCC agreed with John Smith to ship Mr. Smith's furniture from Toronto to Ottawa. Mr. Smith's furniture is worth $100 000.00. Just before loading the furniture onto its truck, HCC's driver had Mr. Smith sign a contract that contained a term that limited HCC's liability for destruction or damage to the furniture to $50.00 if it resulted from the negligence of HCC or its employees. On the way to Ottawa, HCC's driver lost control of the truck and Mr. Smith's furniture was destroyed by fire after the truck crashed into a brick wall. In this situation, a. the limitation of liability in the contract is effective only against HCC, but not its employee, so Mr. Smith can sue the employee for the value of his furniture. b. the limitation of liability in the contract is permitted by law and Mr. Smith will receive only $50.00. c. Mr. Smith's contract with HCC is illegal by common law, so Mr. Smith will recover the full value of his furniture from HCC. d. although the contract with HCC is legal, the limitation on liability contained in it is not, so Mr. Smith will recover the full value of his furniture from HCC. e. the limitation of liability in the contract is not effective, because Mr. Smith did not see it before he signed the contract, so Mr. Smith will recover the full value of his furniture from HCC.

b Diff: 2 Type: MC Topic: Contracts Illegal by the Common Law and Public Policy Skill: Recall/Applied 15

Use this fact situation to answer the related questions that follow. Mary is employed in administration at York University, where she is paid $3500.00 a month. The administrative staff have gone on strike and, to that end, have set up a picket line to prevent employees from going into the university's administration building. Mary wants no part of the strike, so she decides that she will cross the picket line. As she does this, an employee named Ralph says, "Unless you want a black eye, lady, you better not cross the picket line." John, another employee, pushes Mary back. Unknown to Mary, there is a hole in the pavement that the university has been aware of for quite some time, but has done nothing about, and as Mary falls back, her shoe catches in the hole and she falls down, breaking her leg and spraining her right arm. Ralph's statement to Mary is a. the tort of negligence. b. the tort of assault. c. the tort of injurious falsehood. d. the tort of battery. e. the tort of trespass.

b Diff: 2 Type: MC Topic: Intentional Torts Skill: Recall/Applied 5

Use the fact situation in Q3 to answer the related question that follows. John's action in pushing Mary constitutes a. the tort of assault. b. the tort of battery. c. the tort of negligence. d. the tort of injurious falsehood. e. the tort of trespass.

b Diff: 2 Type: MC Topic: Intentional Torts Skill: Recall/Applied 6

The fact that similar cases are treated alike a. increases the number of disputes that go to court. b. reduces the number of disputes that go to court by allowing parties to anticipate the results based on prior outcomes. c. does not help parties in a dispute to anticipate how a case will turn out based on prior outcomes. d. does nothing to reduce the number of disputes that end up in the courts. e. none of the above

b Diff: 2 Type: MC Topic: Legal Systems: Civil Law and Common Law Skill: Recall/Applied 4

When a minor reaches the age of majority, a contract that was entered into earlier and was unenforceable remains unenforceable unless a. the parties execute an affidavit. b. it was ratified after the person attains the age of majority. c. it is repudiated after the person attains the age of majority. d. the subject matter is a non-necessary item. e. the subject matter is stolen before the person attains the age of majority.

b Diff: 2 Type: MC Topic: Minors (or Infants

Jack and Jill are friends. Jack agrees to mow Jill's lawn and Jill agrees to pay Jack $50.00. Jill wants the lawn mowed by April 1. Jill returns on April 3 and finds that the lawn has not been mowed. Jill, who is entertaining the next day, goes to Jack and promises that if he mows the lawn that day, she will give him an additional $20.00. Jack mows the lawn but Jill refuses to pay him the additional $20.00. Which of the following is true? a. Jack can sue Jill for breach of contract. b. Jill has no obligation to pay because the promise to pay the additional $20.00 was not supported by fresh consideration. Jack had a pre-existing legal duty to mow the lawn. c. There was no offer and acceptance. d. Jill has an obligation to pay the additional $20.00 because a legally enforceable promise to pay was made. e. Jill made an offer and Jack accepted it, so a contract was created.

b Diff: 2 Type: MC Topic: Motive Contrasted with Consideration: Past Consideration Skill: Applied 9

Michael is an Aboriginal person living on a reserve. Which of the following is true? a. Michael can use reserve land as security for a loan. b. Michael may manufacture and sell goods to outsiders. c. If Michael moves off the reserve, his capacity to contract would not change. d. Michael has the same capacity to contract as other Canadians. e. Michael's legal position to sell produce would not vary by province.

b Diff: 2 Type: MC Topic: Other Persons of Diminished Contractual Capacity Skill: Applied 19

Mary owes Jack $15 000 in unpaid bills. Mary and Jack enter into an agreement. Jack is required to pay Mary $20 000 for the delivery of certain products. Jack is unable to pay and Mary sues Jack. Under the circumstances, Jack may also a. file a statement of questions. b. counterclaim for $15 000. c. reserve judgment. d. provide admissible evidence. e. request an examination for discovery.

b Diff: 2 Type: MC Topic: Procedural Law: Using the Courts Skill: Applied 27

Joe recently purchased a can of shaving cream from his pharmacy. After shaking the can as directed, Joe pushed the button on top. Suddenly and without warning, the can exploded causing serious injury to Joe. As a result of the explosion, the can of shaving cream was destroyed. In an action by Joe against the manufacturer, the following principle of law will apply: a. The shaving cream manufacturer will be held strictly liable for damages caused by the explosion. b. The court can infer negligence on the part of the manufacturer on the basis of the circumstantial evidence resulting in a shift of the burden onto the manufacturer to prove that it was not at fault. c. Joe will be unable to meet the burden of proof in establishing that the manufacturer was negligent given the destruction of the evidence. d. Joe will be found to be contributorily negligent.

b Diff: 3 Type: MC Topic: Product Liability Skill: Applied 29

Section 1 of the Charter of Rights and Freedoms says that the rights in the Charter are not absolute. Section 1 a. enables Parliament to override decisions of the courts regarding the Charter. b. allows the courts to decide whether it is necessary to infringe on rights in order to maintain the values of a free and democratic society. c. permits the Supreme Court of Canada to amend the Charter to conform to the values of a free and democratic society. d. enables provincial legislatures to override decisions of the courts regarding the Charter. e. none of the above

b Diff: 3 Type: MC Topic: The Charter of Rights and Freedoms Skill: Recall 31

In a federal system such as our own, the role of the courts is to a. draft legislation. b. dispense the law, which includes interpreting it as well. c. advise Parliament as to whether its laws are just and fair. d. ensure that the law is handed out equally to every Canadian. e. make it seem that we have a just legal system, when in fact we do not.

b Diff: 3 Type: MC Topic: The Courts and Legislation Skill: Applied 18

The Supreme Court of Canada performs several roles. One role it does NOT perform is a. protector of civil liberties. b. an umpire between the various levels of government. c. determining the validity of legislation. d. interpreter of legislation. e. resolving disputes between private parties.

b Diff: 3 Type: MC Topic: The Courts and Legislation Skill: Applied 22

Whenever a law is determined by a court to be outside the jurisdiction of the legislature, and beyond its powers, the law or provision is said to be a. a federal law. b. ultra vires and therefore void. c. the result of residual power. d. excess of jurisdiction and therefore invalid. e. the result of concurrent powers.

b Diff: 3 Type: MC Topic: The Courts and Legislation Skill: Recall 24

On June 1 George goes to Harry's fruit store, and while he is there, Harry offers to sell George 100 bushels of grapes that have just been delivered to his store for $20.00 per bushel. Harry also tells George that he should not wait too long to accept Harry's offer, because the best time to sell grapes is within three to four days of delivery. George simply says, "Let me think about it and I'll get back to you before the end of the day." However, George does not return that day; instead, he comes back seven days later to tell Harry that he accepts the offer. Harry replies, "You're much too late, I sold the grapes to Mr. Smith on June 5." In this situation a. George's statement "Let me think about it and I'll get back to you before the end of the day" is a counter-offer. b. Harry's offer has lapsed given the perishable nature of the grapes. c. Harry was wrong in selling the grapes to Mr. Smith. d. Harry must keep his offer open until George accepts it. e. Harry's offer has been withdrawn.

b Diff: 3 Type: MC Topic: The Lapse and Revocation of an Offer Skill: Recall/Applied 18

Use the fact situation in Q6 to answer the related question that follows. In this situation, the bank's conduct in giving the erroneous financial statement to Carol constituted a. a breach of contract only. b. a negligent misrepresentation. c. a fraudulent misrepresentation. d. negligence only. e. neither negligence, no breach of contract or negligent misrepresentation.

b Diff: 3 Type: MC Topic: Tort Liability for Inaccurate Statements Skill: Applied 9

The difference between a negligent misrepresentation and a fraudulent misrepresentation is that a. a fraudulent misrepresentation requires only a breach of duty of care and skill, while a negligent misrepresentation requires at least some guilty knowledge or willful disregard of the falsity of information provided. b. a negligent misrepresentation requires only a breach of duty of care and skill, while a fraudulent misrepresentation requires at least some guilty knowledge or willful disregard of the falsity of information provided. c. there is no practical difference between a fraudulent misrepresentation and a negligent misrepresentation d. a fraudulent misrepresentation does not require guilty knowledge or willful disregard of the falsity of information provided, while a negligent misrepresentation does. e. a negligent misrepresentation does not require a breach of duty of care and skill, while a fraudulent misrepresentation does.

b Diff: 3 Type: MC Topic: Tort Liability for Inaccurate Statements Skill: Recall/Applied 3

Bilateral contracts are usually made up of two stages - promises by both parties to perform and later, performance or fulfillment of the promises. When one of the parties to a bilateral contract does not receive the performance as promised a. a cause of action is recognized at law giving the promisee the right to sue for tort damages. b. a cause of action is recognized at law giving the promisee the right to sue for damages for breach of contract. c. no cause of action is recognized at law until the promisee satisfies the court that the damages have been mitigated. d. no cause of action is recognized at law until all possible criminal proceedings are disposed of. e. no action can be brought against any party for simply failing to fulfill its contractual promises.

b Diff: 3 Type: MC Topic: Unilateral and Bilateral Contracts Skill: Applied 24

Stoney Canyon penitentiary is located near the town of Elmwood. One evening a guard employed at the jail failed to properly secure one of the gates near an exercise field. As a result, an inmate escaped and went on a crime spree. Having regard to issues of proximity, foreseeability, and policy reasons, which of the following claimants would be least likely to succeed in a court action against the guard and/or the jail? a. A claim by a citizen of Elmwood whose vehicle was carjacked within hours of the escape. b. A claim by a gas station owner for personal injury and economic losses occasioned by the escaped inmate's assault and robbery which took place two months later and 250 kilometres away from the jail. c. The claim of a local bingo parlour whose premises were torched by the inmate after a standoff with the police the day after his escape. d. The claims of an elderly couple injured in an automobile accident two days after the escape during which the inmate crashed into their car following a high speed pursuit.

b Diff: 3 Type: MC Topic: Unintentional Torts Skill: Applied 30

Joe sees a dog wandering around his yard. He lets the dog in. Later he learns that the dog belongs to Mary, a woman who lives a kilometre away. Joe returns the dog to her the next day. A few days later as Joe is riding his bike, he sees an old advert in which Mary offers to pay $100 to any person who will find and return her dog. Joe realizes that he should have claimed the reward at the time that he returned the dog to Mary, and he decides to go to Mary's place. Which of the following is true? a. Joe will be entitled to the reward because he found the dog and returned it. b. Joe will not be entitled to the reward because Mary made no offer. c. Joe will not be entitled to the reward because he did not see the offer of the reward until he had returned the dog and so he did not act in response to the offer of Mary. d. Mary's advert is an invitation to do business and cannot give rise to a contract. e. Joe will be entitled to the reward because Mary made an offer and his act of finding and returning the dog is an acceptance.

c Diff: 2 Type: MC Topic: The Elements of Acceptance Skill: Applied 9

Use the fact situation in Q3 to answer the related question that follows: In the above fact situation, an offer occurs a. when Alexander takes the tea home. b. when the cashier tells Alexander the price of the tea. c. when Alexander hands the money to the cashier. d. when Alexander sees the price beside the teas on the shelves. e. when Alexander takes the tea off the shelf.

c Diff: 2 Type: MC Topic: The Nature of an Offer Skill: Recall/Applied 5

Which of the following is NOT one of the tiers of courts in Canada? a. intermediate provincial appeal courts b. Supreme Court of Canada c. circuit courts d. court of first instance e. none of the above

c Diff: 2 Type: MC Topic: The System of Courts in Canada Skill: Recall 25

Use the fact situation in Q3 to answer the related question that follows. The failure of the university to repair the hole in the pavement constitutes a. the tort of nuisance. b. the tort of trespass. c. the tort of negligence. d. the tort of injurious falsehood. e. the tort of battery.

c Diff: 2 Type: MC Topic: Unintentional Torts Skill: Recall/Applied 7

The most common complaint about judges when they are interpreting statutes or legislation is that they a. are much too powerful and have no regard for anyone other than themselves. b. are usurping the powers of Parliament and the provincial legislatures. c. are too political and are usurping the powers of Parliament and the provincial legislatures. d. have no regard for the average citizen. e. are too political.

c Diff: 3 Type: MC Topic: Challenging the Validity of a Statute Skill: Applied 19

Mo agrees to provide lawn care services to Judith in preparation for a summer birthday party Judith is holding for family and friends. The agreement requires that Judith pay Mo $250.00 for three hours' work. After completing approximately half of the job, Mo requests that Judith pay him an extra $50.00 because the cost of gasoline used for the rider mower had gone up significantly that morning. Judith agrees to pay the additional $50.00. Is she legally liable to do so? a. Yes she is, because she told Mo she would pay him and he finished the job on the basis of her promise. b. Yes she is, because the rising price of gas should be borne by Judith. c. No she isn't, because Mo had an existing contractual obligation to perform the services for $250.00 and there is no consideration for the subsequent agreement to pay $50.00 more. d. No she isn't, because Judith is promising to reward Mo for something he has previously done and the past consideration is no consideration.

c Diff: 3 Type: MC Topic: Existing Legal Duty and Consideration Skill: Applied 30

A, at the request of B, enters into a contract to buy a horse from B for a certain price. Just after entering into the contract, B tells A that the horse is sound and healthy. Later A discovers that, in fact, the horse is in ill health. In this situation, B's statement to A that the horse was sound and healthy a. is partly fresh and past consideration so as to not make B's promise binding. b. is partly fresh and partly past consideration so as to make B's promise binding. c. is past consideration so as to not make B's promise binding d. is fresh consideration so as to make B's promise binding e. all of the above

c Diff: 3 Type: MC Topic: Gratuitous Promises Skill: Recall/Applied 15

Albert and Jane and her husband own homes beside each other. Jane has a swimming pool in her back yard. Between the houses, on Albert's property, there is a line of very old trees that have grown so tall that they are blocking the sunlight from the swimming pool area of Jane's property. Yesterday, Jane's husband, George, after hearing Jane complain about the blocked sunlight, without going onto Albert's property, cut 3 metres off the top of each of Albert's trees to let the sunlight into his and Jane's back yard. George's action in cutting down the trees constitutes a. the tort of defamation. b. the tort of assault. c. the tort of nuisance. d. the tort of trespass. e. the tort of negligence.

c Diff: 3 Type: MC Topic: Intentional Torts Skill: Applied 4

The Canadian legal profession is organized on a provincial basis with each province having its own "bar" society. However, a member of any provincial bar may appear before a. the House of Lords in England. b. the International War Crimes Tribunal. c. the Supreme Court of Canada. d. any Human Rights Tribunal in any province. e. any Landlord and Tenant Board Tribunal in any province.

c Diff: 3 Type: MC Topic: Legal Risk Management Skill: Recall 5

The purchaser of a bottled beverage is more likely to be successful in a suit for damages against the manufacturer as a result of becoming ill if a. she saw a long black hair in the bottle and decided not to drink it. b. she saw a long black hair in the bottle and dropped it on the ground before she opened it. c. she saw a long black hair in the bottle after taking "a sip." d. she gave the bottle to a friend and saw a long black hair in the bottom after her friend drank it.

c Diff: 3 Type: MC Topic: Product Liability Skill: Applied 34

Brayden had a long history of intestinal disorder. After several failed non-invasive treatments, it was suggested to him that he undergo a colonoscopy as a means of better diagnosing his problem. He spent five minutes prior to the procedure consulting with the doctor who performed the colonoscopy. He was told by the doctor that these procedures usually are simple and for the most part problem-free. Brayden agreed to undergo the procedure. During the course of the colonoscopy his bowel was perforated and he was required to undergo a significant open surgical procedure to save his life. Brayden sued the doctor for lack of informed consent. At trial, the expert evidence established that the risks of bowel perforation were material risks and as such ought to have been disclosed. Brayden in fact testified that had he been told of the risk of bowel perforation he would have not undergone the colonoscopy. However, the judge hearing the case made a finding that a reasonable person with Brayden's condition having been properly informed of all the risks would have undergone the colonoscopy. In these circumstances, the judge will likely then proceed to a. allow the action because of lack of informed consent. b. allow the action because of the fact that Brayden himself testified he would not have undergone the procedure had he been told of the risks of bowel perforation. c. dismiss the action because a reasonable person in the position of the plaintiff would have proceeded with the colonoscopy had they been informed of the materials risks. d. dismiss the action because there is no causation. e. allow the action because Brayden had to undergo an unexpected and unanticipated life-saving surgery.

c Diff: 3 Type: MC Topic: Proving Negligent Misrepresentation Skill: Applied 29

On January 1, A offers to sell his property to B for $175 000.00 and gives B an option to purchase the property until July 1. The option reads: "The said option shall be exercisable by notice in writing to the seller at any time within 6 months from the date of the option." B mails his acceptance to A on February 12, but the letter never reaches A. In this situation, a. B's acceptance by mail is not in accordance with the option and there is no contract between A and B. b. since the option states that it is exercisable by notice in writing to the seller, mailing the acceptance is deemed to be notice to the seller. c. since the option states that it is exercisable by notice in writing to the seller, and since the acceptance never came to the seller, B's acceptance by mail is not acceptance at all. d. B's acceptance of the option by mail is a proper acceptance, creating a contract between A and B. e. none of the above

c Diff: 3 Type: MC Topic: The Elements of Acceptance Skill: Applied 26

On October 1, Allan mailed an offer to Bob to sell to Bob 1000 boxes of tin plates at a fixed price. The offer was received by Bob on October 11, and Bob immediately accepted the offer by telegram on October 11 and confirmed by letter on October 15. On October 8, Allan mailed a revocation of his offer, which was received by Bob on October 20. In this situation a. Allan's revocation is really a counter-offer. b. Allan has no right to try to revoke his offer. c. Allan's revocation comes too late and there is a contract between Allan and Bob. d. Allan's revocation of his offer is proper and effective to prevent a contract between Allan and Bob. e. none of the above

c Diff: 3 Type: MC Topic: The Elements of Acceptance Skill: Recall/Applied 17

Federalism is a. the system of government that applies in the Province of Quebec. b. the system of government in which the central government makes laws for all the people. c. the system of government in which power is divided between two levels of government. d. a system of government in which the various governments, such as federal and provincial, all come together to make laws for all the people. e. the system of government that believes in health care for all.

c Diff: 3 Type: MC Topic: Who Makes Law? Skill: Recall 25

Use the fact situation in Q23 to answer the related question that follows. Review the fact situation above and now assume that instead of simply promising to pay B an extra $5000.00, A says, "I'll pay you an extra $5000.00 if you fix that cracked board in my fence. Also assume that B completes the job by September 1 and fixes the cracked board. In this situation, a. there is still no fresh consideration from B, so A does not owe the extra $5000.00. b. since B completed the contract on September 1st, B is only entitled to the original price of $60 000.00. c. the contract did not change on June 2, so A does not owe the extra $5000.00. d. the promise to fix the cracked board and the fixing of it is fresh consideration and B is entitled to the extra $5000.00. e. the promise to fix the cracked board and the fixing of it was a gratuitous promise so B is not entitled to the extra $5000.00.

d Diff: 2 Type: MC Topic: Adequacy of Consideration Skill: Applied 26

Which of the following is generally used to resolve an international dispute? a. mediation b. litigation c. settlement d. arbitration e. alternative dispute resolution

d Diff: 2 Type: MC Topic: Alternative Dispute Resolution Skill: Recall 16

In our legal system, the decision of a court of first instance a. can only be overruled by the Supreme Court of Canada. b. can only be overruled by a similar court of first instance. c. can only be overruled by a similar court of first instance and an appeal court. d. can only be overruled by either an appeal court or the Supreme Court of Canada. e. can only be overruled by an appeal court.

d Diff: 2 Type: MC Topic: Common Law: The Theory of Precedent Skill: Applied 8

Which of the following is NOT true? a. A corporation has no physical existence. b. A corporation is a legal fiction. c. A corporation is a creature of law. d. A corporation does not have the attributes of a natural person. e. A corporation has the same attributes as a natural person.

d Diff: 2 Type: MC Topic: Corporations Skill: Recall 11

Which of the following is true regarding the legal capacity of the corporation? a. Agents can negotiate but not sign contracts on behalf of the corporation. b. All contracts need to be placed under the corporate seal. c. The corporation can show it is ultra vires to have contracts enforceable. d. Authorized officers of the corporation can sign most contracts. e. Neither statute nor incorporating documents can limit the capacity.

d Diff: 2 Type: MC Topic: Corporations Skill: Recall 25

A promise by a creditor to accept less than full payment from a debtor is a. unenforceable because the creditor did not provide consideration. b. enforceable because the promisee accepts the offer. c. a bilateral contract. d. unenforceable because it is a gratuitous promise. e. enforceable because it gives rise to a contract.

d Diff: 2 Type: MC Topic: Gratuitous Reduction of a Debt Skill: Recall 22

Jack walks into a bar. He is a little tipsy. The waitress pours him a pint of beer and he pays with all the money he has on him. He makes this statement to the waitress: "Sorry I can't tip you today, but you know what, I have this lottery ticket, and if I win, I'll split it with you." The waitress says, "No problem, Jack." Jack later makes a copy of the ticket for the waitress. The next day Jack receives a call from Stephanie, the waitress, who tells him that she checked the numbers and that he has won $1 million. Stephanie asks, "Our deal is on right?" Jack says, "What deal?" She says, "You promised to split the winning ticket with me." He says, "Really?" Which of the following is true? a. Jack will have to split the proceeds with Stephanie on a quantum meruit basis. b. Stephanie can sue Jack for breach of contract if she does not get her share of the ticket. c. Jack promised to split the winning ticket with Stephanie and she accepted, so a valid contract was created. d. No contract arose because there was no intention on the part of Jack to enter into legal relations with Stephanie. e. A contract did not arise because Jack's statement was not an offer.

d Diff: 2 Type: MC Topic: Intention to Create Legal Relations Skill: Applied 18

Ted is employed as a computer analyst at Macrotech. His written contract of employment requires him to give his employer 30 days' notice of his intention to terminate the agreement. Macrotech's main competitor, Data Inc., attempts to hire Ted away. Data Inc.'s president convinces Ted to accept their employment offer and to leave Macrotech immediately, without providing any notice pursuant to the agreement. Data Inc. has committed the tort of a. passing off. b. negligence. c. malicious prosecution. d. inducing breach of contract.

d Diff: 2 Type: MC Topic: Intentional Torts Skill: Recall 16

Harvey is a teacher. He is also a member of a labour union that represents teachers. For the past month, the union has been negotiating a new agreement with the government of Ontario. Yesterday, the union executive authorized Harvey to bring a lawsuit against the government of Ontario for failure to bargain in good faith. In this situation, a. the labour union is just like a corporation and can bring a lawsuit against the government. b. only Harvey himself can bring a lawsuit against the government, not the union. c. just like a social club, the labour union cannot bring a lawsuit against the government. d. the labour union can appoint Harvey to bring a representative action on behalf of the union against the government. e. the law is uncertain as to the status of labour unions to bring a lawsuit against the government.

d Diff: 2 Type: MC Topic: Labour Unions Skill: Recall/Applied 27

A binding code of conduct is a a. general code of conduct that is available to anyone. b. code of conduct established voluntarily by industry. c. code of conduct that governments impose on business and industry. d. code of conduct that regulates specified activities, such as those of professionals. e. code of conduct that a firm or business imposes on itself.

d Diff: 2 Type: MC Topic: Law and Business Ethics Skill: Recall 14

Damages for breach of fiduciary duty is restitutionary in that a. the breach is not a tort. b. the breach is not a breach of contract. c. the client may be contributorily negligent. d. the client is entitled to be placed in the position he or she would have been had the breach not occurred. e. the professional is required to give back what she or he has taken from the client.

d Diff: 2 Type: MC Topic: Liability of Professionals Skill: Applied 23

Alice Walters is a patient of Dr. John Good, a heart surgeon. Dr. Good had just begun to tell Alice that her treatment wasn't working and that she would need to have surgery to correct a defect in her heart, but then he was called away on an emergency and forgot to return to his conversation with Alice and explain all of the risks of the surgery to her. In fact, Dr. Good had diagnosed Alice's condition three years ago after returning to work from his office Christmas party. He had had a number of alcoholic beverages, and this caused him to misinterpret a lump on her lung for a lump on her heart. After diagnosing Alice, Dr. Good had told Alice that the problem was with her heart and then prescribed treatment for it. Relying on what Dr. Good had told her, Alice had accepted the treatment, which was very painful. In this situation, it can be said that a. Dr. Good made a negligent misrepresentation about the nature of Alice's problem and the treatment for it. b. Dr. Good was negligent in his diagnosis. c. Dr. Good failed to obtain from Alice an informed consent. d. Dr. Good committed a breach of contract in misdiagnosing Alice's condition. e. all of the above

e Diff: 3 Type: MC Topic: Tort Liability for Inaccurate Statements Skill: Applied 6

Use the fact situation in Q3 to answer the related question that follows. If Mary wants to show that the university was negligent, Mary must prove a. that the injuries were a foreseeable result of the failure to repair the hole in the pavement. b. that the university breached its duty to repair the hole in the pavement. c. that Mary suffered injuries as a result of the failure to repair the hole in the pavement. d. that the university owed her a duty to repair the hole in the pavement. e. all of the above

e Diff: 3 Type: MC Topic: Unintentional Torts Skill: Recall/Applied 9


Conjuntos de estudio relacionados

Chapter 24, The Child with Renal Dysfunction

View Set

Intro to Programming Concepts Midterm Study Guide

View Set

дієслово як частина мови

View Set

NUR 106 Module A Practice Questions

View Set